Win up to 100% Scholarship

Register Now

Modern History

 

2022

 

Question 1

With reference to Indian history, consider the following statements:

  1. The Dutch established their factories/warehouses on the east coast on lands granted to them by Gajapati rulers.

  2. Alfonso de Albuquerque captured Goa from the Bijapur Sultanate.

  3. The English East India Company established a factory at Madras on a plot of land leased from a representative of the Vijayanagara empire.

Which of the statements given above are correct?

(a) 1 and 2 only

(b) 2 and 3 only

(c) 1 and 3 only

(d) 1, 2 and 3

Ans: b

Sub-Theme: Various events during the advent of the Europeans

  • Statement 1 is incorrect: There is a difference in timeline of the events. The end of Gajapati Dynasty was in 1542 and the United East India Company of the Netherlands, formed in March 1605 by the Charter of Dutch Parliament and founded their first factory in Masulipatnam in Andhra in 1605.
  • Statement 2 is correct: Alfonso de Albuquerque considered as Founder of the Portuguese power in India: captured Goa from Bijapur; persecuted Muslims; captured Bhatkal from Sri Krishna Deva Rai (1510) of Vijayanagara.
  • Statement 3 is correct: The English East India Company established a factory in Madras in 1639 on land leased from representatives of Vijayanagara Empire called the Nayakas

 

Question 2

Consider the following freedom fighters:

  1. Barindra Kumar Ghosh

  2. Jogesh Chandra Chatterjee

  3. Rash Behari Bose

Who of the above was/were actively associated with the Ghadar Party?

(a) 1 and 2

(b) 2 only

(c) 1 and 3

(d) 3 only

Ans: d

Sub-Theme: Revolutionary Activities Abroad

Ghadar Party, 1913:

  • Ghadar Party was a revolutionary group organised around a weekly newspaper “The Ghadar” with its headquarters at San Francisco and branches along the US coast and in the Far East.
  • These revolutionaries included mainly ex-soldiers and peasants who had migrated from the Punjab to the USA and Canada in search of better employment
  • Pre-Ghadar revolutionary activity had been carried on by Ramdas Puri, G.D. Kumar, Taraknath Das, Sohan Singh Bhakna and Lala Hardayal who reached there in 1911.
  • To carry out revolutionary activities, the earlier activists had set up a ‘Swadesh Sevak Home’ at Vancouver and ‘United India House’ at Finally, in 1913, the Ghadar was established, with the aim of organising assassinations of officials, publishing revolutionary and anti- imperialist literature, work among Indian troops stationed abroad, procure arms and bring about a simultaneous revolt in all British colonies.
  • The moving spirits behind the Ghadar Party were Lala Hardayal, Ramchandra, Bhagwan Singh, Kartar Singh Saraba, Barkatullah and Bhai Other prominent leaders involved in the movement: Kartar Singh Saraba, Raghubar Dayal Gupta, Rash Behari Bose, and Sachin Sanyal.

Additional Information:

  • Barindra Kumar Ghosh was associated with a revolutionary group called Jugantar and Jogesh Chatterjee was associated with the Hindustan Republican Association/Army (HRA).

 

Question 3

With reference to the proposals of Cripps Mission, consider the following statements:

  1. The Constituent Assembly would have members nominated by the Provincial Assemblies as well as the Princely States.

2. Any Province, which is not prepared to accept the new Constitution would have the right to sign a separate agreement with Britain regarding its future status.

Which of the statements given above is/are correct?

(a) 1 only

(b) 2 only

(c) Both 1 and 2

(d) Neither 1 nor 2

Ans: b

Sub-Theme: Cripps Mission

Cripps Mission (1942):

  • In March 1942, a mission headed by Stafford Cripps was sent to India with constitutional proposals to seek Indian support for World War Because of the reverses suffered by Britain in South-East Asia, the Japanese threat to invade India seemed real now and Indian support became crucial.
  • There was pressure on Britain from the Allies (USA, USSR, and China) to seek Indian

Provisions:

  • Indian union with a Dominion status would be set up and it would be free to decide its relation with the UN and the Commonwealth.
  • After the war the Constituent Assembly would be formed with members from Provinces (elected through proportional representation) and Princely states (nominated). (Hence, statement 1 is )
  • Conditions of Accepting New Constitution:
    • Any Province not willing to join the Union can have a separate Constitution and form a Separate union. (Hence, statement 2 is correct.)
    • New Constitution making Body and the Government would negotiate a treaty for Transfer of Power.

 

2021

 

Question 1

In the first quarter of the seventeenth century, in which of the following was/were the factory/factories of the English East India Company located?

  1. Broach

  2. Chicacole

  3. Trichinopoly

Select the correct answer using the code given below.

(a) 1 only

(b) 1 and 2

(c) 3 only

(d) 2 and 3

Ans: a

Sub-Theme: Factories of English East India Company

The English East India Company had a factory at Broach (present-day Bharuch) in Gujarat, India during the seventeenth century. However, Chicacole (present-day Srikakulam) and Trichinopoly (present-day Tiruchirappalli) were not among the locations of the Company’s factories during this period. Establishment of Factories by English East India Company:

  • West Coast: The English India Company established their factories at Agra, Ahmadabad, Baroda and Broach by 1619, under the control of the president and council of the Surat factory which was established in the year 1612.
  • South-eastern Coast: EIC established their factories at Masulipatam in 1611 and Armagaon near Pulicat in 1626.
  • Eastern India: In eastern India EIC established their factories at Hariharpur and Balasore(now Odisha) in 1633, at Hugli in 1651, followed by setting up factories at Patna, Dacca, Kasimbazar in Bengal and Bihar respectively.
NOTE: This is a direct question asked from Spectrum Modern India (revised edition)

 

Question 2

Consider the following statements:

  1. The Montagu-Chelmsford Reforms of 1919 recommended granting voting rights to all the women above the age of 21.

  2. The Government of India Act of 1935 gave women reserved seats in the legislature.

Which of the statements given above is/are correct?

(a) 1 only

(b) 2 only

(c) Both 1 and 2

(d) Neither 1 nor 2

Ans: b

Sub-Theme: Administrative Reforms post 1857

Montagu-Chelmsford Act, 1919:

  • By demarcating and separating the central and provincial subjects, it relaxed the central control over the provinces
  • Divided the provincial subjects into two parts:
    • Transferred and Reserved subjects: This dual scheme of governance was known as ‘dyarchy’.
  • Introduced bicameralism and direct elections in the country.
  • It required 3 out of 6 members in the Viceroy’s Executive Council to be Indian.
  • Recommended the voting rights to women in limited numbers to be extended on the basis of property, tax or education
  • Extended the principle of communal representation among the Sikhs, Indian Christians, Anglo-Indians and Europeans.

Government of India Act of 1935 Features:

  • Act divided the powers between the Centre and units in terms of three listsFederal List (for Centre, with 59 items), Provincial List (for provinces, with 54 items) and the Concurrent List (for both, with 36 items). Residuary powers were given to the Viceroy.
    • However, the federation never came into being as the princely states did not join it.
  • It abolished dyarchy in the provinces and introduced ‘provincial autonomy’ in its place. The provinces were allowed to act as autonomous units of administration in their defined spheres.
    • Moreover, the Act introduced responsible governments in provinces, that is, the governor was required to act with the advice of ministers responsible to the provincial legislature. This came into effect in 1937 and was discontinued in 1939.
  • It provided for the adoption of dyarchy at the Centre.
    • Consequently, the federal subjects were divided into reserved subjects and transferred subjects.
    • However, this provision of the Act did not come into operation at all.
  • It introduced bicameralism in six out of eleven provinces. Thus, the legislatures of Bengal, Bombay, Madras, Bihar, Assam and the United Provinces were made bicameral consisting of a legislative council (upper house) and a legislative assembly (lower house).
  • It extended the principle of separate electorates for depressed classes (scheduled castes), women and labour (workers).
  • It abolished the Council of India, established by the Government of India Act of 1858. The secretary of state for India was provided with a team of advisors.
  • Extended franchise: About 10 per cent of the total population got the voting right.
  • It provided for the establishment of a Reserve Bank of India to control the currency and credit of the country.
  • It provided for the establishment of not only a Federal Public Service Commission but also a Provincial Public Service Commission and Joint Public Service Commission for two or more provinces.

 

Question 3

With reference to 8th August 1942 in Indian history, which one of the following statements is correct?

(a) The Quit India Resolution was adopted by the AICC.

(b) The Viceroy’s Executive Council was expanded to include more Indians.

(c) The Congress ministries resigned in seven provinces.

(d) Cripps proposed an Indian Union with full Dominion Status once the Second World War was over.

Ans: a

Sub-Theme: Quit India Movement

Option (a) is correct: Quit India Resolution was ratified in a Congress meeting at Gowalia Tank on 8th August 1942. Apart from QIM, other resolutions involved the Civil Disobedience Movement against British Rule. Mantra of “Do or Die” was adopted in this Session by Gandhiji. Option (b) is incorrect: More Indians were added to Viceroy’s executive council by the Government of India Act 1919. First time, an Indian was added in this council by the Government of India act 1909.

Option (c) is incorrect: The Congress Ministries resigned from seven provinces in 1939 in protest because the then Viceroy Lord Linlithgow’s (1936-1944) action declared India to be belligerent in the WW-II without even consulting the Indian people.

Option (d) is incorrect: Though Cripps proposed a dominion status to India and the failure of the cripps mission resulted in a nationwide Quit India Movement, this was not related to 8 August 1942.

 

Question 4

In the context of Colonial India, Shah Nawaz Khan, Prem Kumar Sehgal and Gurbaksh Singh Dhillon are remembered as:

(a) leaders of Swadeshi and Boycott Movement

(b) members of the Interim Government in 1946

(c) members of the Drafting Committee in the Constituent Assembly

(d) officers of the Indian National Army

Ans: d

Sub-Theme: Indian National Army/Netaji Subhash Chandra Bose/Red Fort Trial

Indian National Army (INA) Trials/ Red Fort Trials:

  • Between November 1945 and May 1946, the INA officers were brought before a court martial at the Red Fort in Delhi. At the Red Fort in Delhi, about ten court-martials were held in public.
  • By holding open trials in the Red Fort, Claude Auchinleck, the commander-in-chief of the British-Indian army, hoped to sway public opinion against the INA.
  • Major General Shah Nawaz Khan, Colonel Prem Kumar Sahgal and Colonel Gurbaksh Singh Dhillon were three of the senior-most officers of INA and trusted colleagues of Netaji.
  • After considerable demonstrations and unrest in India, they were forced to be released after the British court-martialed them at Red Fort in 1945 and sentenced them to death.
  • Congress leader and the country’s first Prime Minister, Jawaharlal Nehru, also got on board the INA officers’ legal defence team, along with party colleague Bhulabhai Desai and barrister Tej Bahadur Sapru.
NOTE: As we have already discussed, the trend of personality based questions continues !!

 

Question 5

Who among the following was associated as Secretary with Hindu Female School which later came to be known as Bethune Female School?

(a) Annie Besant

(b) Debendranath Tagore

(c) Ishwar Chandra Vidyasagar

(d) Sarojini Naidu

Ans: c

Sub-Theme: Reforms in Education during the British Time

Ishwar Chandra Vidyasagar (1820-1891) was appointed as the secretary of Bethune School/College established in 1849. Bethune School began as Hindu Female School in 1849 was renamed Bethune School in 1856. In 1856, the Government took charge of the Hindu Female School, later renamed Bethune School.

NOTE: Details of Annie Besant has already been covered in PYQs under Prominent Personalities in Modern History.

 

Question 6

With reference to Madanapalle of Andhra Pradesh, which one of the following statements is correct?

(a) Pingali Venkayya designed the tricolour Indian National Flag here.

(b) Pattabhi Sitaramaiah led the Quit India Movement of Andhra region from here.

(c) Rabindranath Tagore translated the National Anthem from Bengali to English here.

(d) Madame Blavatsky and Colonel Olcott set up headquarters of Theosophical Society first here.

Ans: c          

Sub-Theme: Major events/Major places of Indian

Freedom Struggle

Option (a) is incorrect: The Indian tricolour was designed by Pingali Venkayya, who was a freedom fighter and was a follower of Mahatma Gandhi. Pingali Venkayya had designed the flag of India and presented it to Mahatma Gandhi in 1921 during the session of the All India Congress Committee in Vijayawada.

Option (b) is incorrect: During the QIM in Andhra the Provincial Congress Committee had issued a circular popularly known as the ‘Kurnool Circular’ as the police seized the copy when they rode ‘Kurnool Congress Office. This was drafted by Kala Venkat Rao, on 29th July 1942 and was sent for the approval of the Congress Working Committee through Dr. Pattabhi Sitaramaiah, a member of the working committee.

Option (c) is correct: Rabindranath Tagore translated the National Anthem from Bengali to English here.

Option (d) is incorrect: The Theosophical Society was founded by Madame H. P. Blavatsky and Colonel Olcott in New York in 1875. In 1882, the headquarters of the Society were established in Adyar, near Madras (now Chennai) in India.

 

Question 7

Who among the following is associated with ‘Song from Prison’, a translation of ancient Indian religious lyrics in English?

(a) Bal Gangadhar Tilak

(b) Jawaharlal Nehru

(c) Mohandas Karamchand Gandhi

(d) Sarojini Naidu

Ans: c        

Sub-Theme: Important Personalities/Phases of Mahatma Gandhi

Quit India Movement planned an all-out campaign to compel British withdrawal from India, after the failure of the Cripps Mission to reach a compromise. At the historic August meeting at Gowalia Tank in Bombay, Gandhi proclaimed his mantra—’do or die’. He was arrested on August 9, 1942. He undertook a 21-day fast in February 1943 to protest against the Government actions against Indians involved in the movement. Mahatma Gandhi wrote “Songs of Prison” during his imprisonment in Yerwada Jail, Poona. He translated hymns from the Upanishads and poems by Indian saint poets into English and that collection was published as Songs from the prison.

 

2020

 

Question 1

With reference to the history of India, consider the following pairs:

  1. Aurang – In-charge of treasury of the State

  2. Banian – Indian agent of the East India Company

  3. Mirasidar – Designated revenue payer to the State

Which of the pairs given above is/are correctly matched?

(a) 1 and 2 only

(b) 2 and 3 only

(c) 3 only

(d) 1, 2 and 3

Ans: b

Sub-Theme: Important Keyword related to Colonial Revenue Policy

  • Pair 1 is incorrect: Aurang is a Persian term referring to a warehouse where goods are collected before being sold.
  • Pair 2 is correct: The word Bania (also Vania) is derived from the Sanskrit vanij, meaning ‘a merchant’. The banian acted as an agent for individual East India Company managers and performed a range of functions including, bookkeeping, interpreting, and brokerage, as well as managing his household and personal business activities. Banians served as middlemen for European merchants in Bengal during the British era.
  • Pair 3 is correct: Mirasdar in Urdu means hereditary landowner, co-proprietor and in Persian, miras is inheritance, estate. Under the Ryotwari settlement system in Southern India, the East India Company recognized mirasdars as the only proprietors of land, dismissing tenants’ rights completely. They were prohibited by law from selling their land but it can be leased to tenants for a nominal fee in return for its cultivation. They acted as a designated revenue payer of the state.

 

Question 2

Which of the following statements correctly explain the impact of the Industrial Revolution on India during the first half of the nineteenth century?

(a) Indian handicrafts were ruined.

(b) Machines were introduced in the Indian textile industry in large numbers.

(c) Railway lines were laid in many parts of the country.

(d) Heavy duties were imposed on the imports of British manufactures.

Ans: a

Sub-Theme: Impact of Industrial Revolution on indigenous industry

Declining and deindustrialisation of the Indian traditional cottage and handicraft industry was a direct effect of the industrial revolution in England along with the colonisation of India. Industrial Revolution and its impact on India’s indigenous industry

  • The Industrial Revolution brought severe consequences to Indian society.
  • The fall and destruction of the urban and rural handicraft industry was one of the most significant effects of British Rule..
  • The Industrial Revolution in England also affected the textile industries in India in many ways, such as:
    • English textiles were now posing a serious threat to Indian textiles in the European and American markets.
    • Textiles from India were subject to high taxes in Britain.
    • The African markets were successfully taken over by cheap, machine-made textiles from Britain.
    • By the 1830s, inexpensive British manufactured products had inundated the Indian marketplaces. The Indian textile industries suffered as a result of them being less expensive than Indian textiles.
  • Following the Industrial Revolution, Britain began making textiles by a machine that were considerably more affordable and of higher quality than conventional Indian textile.
  • The start of the industrial revolution had a negative impact on the Indian handicraft in India.
  • Additionally, the decline of the kings, chieftains, and zamindars who served as the employers of these masters of handicrafts led to the steady expansion of British dominion in India through outright battles with the rulers, which resulted in the suffering of handicrafts in India.
  • India became both a raw material supplier and a finished goods market. India, a country based on agriculture, became an economic colony.
  • The British items were inexpensive and only required a little amount of duty, while the Indian goods were subject to heavy import duties.

 

Question 3

Indigo cultivation in India declined by the beginning of the 20th century because of

(a) peasant resistance to the oppressive conduct of planters

(b) its unprofitability in the world market because of new inventions

(c) national leaders’ opposition to the cultivation of indigo

(d) Government control over the planters

Ans: b

Sub-Theme: Peasant Movement

  • Indigo cultivation in India declined by the beginning of the 20th century because of new inventions of improved versions of chemical colours that substitute indigo, thus the production of indigo became unprofitable in the world market. Thus, its unprofitability in the world market because of new inventions’ caused decline in the Indigo Cultivation.

Indigo Revolt 1856-57:

  • Area: Bengal
  • Leaders: Bishnu Charan Biswas and Diagambar Biswas
  • The great worth of indigo as a commerce commodity led to its frequent usage as the nickname “blue ” It gives the materials a blue tint.
  • Reason: They started losing their land due to poor output and they started fall into debt trap
  • India’s first peasant strike resulted in the compelled planting of indigo since the trade in it was profitable because blue dye was in high demand.
  • During their strike, peasants refused to produce indigo or sign The strike expanded to other parts of Bengal, and other zamindars supported the uprisings. A commission was established in their favour.
  • Intelligentsia helped to make it a powerful
  • Harsh Chandra Mukherjee (editor Hindu patriot), Dinabandhu Mitra play: Neel Darpan.

 

Question 4

With reference to the history of India, “Ulgulan” or the Great Tumult is the description of which of the following events?

(a) The Revolt of 1857

(b) The Mappila Rebellion of 1921

(c) The Indigo Revolt of 1859 – 60

(d) Birsa Munda’s Revolt of 1899-1900

Ans: d

Sub-Theme: Tribal Movement

Munda Rebellion:

  • It is one of the prominent 19th century tribal rebellions in the subcontinent. In the years 1899–1900, Birsa Munda served as the movement’s leader in the south of
  • The Ulgulan, also known as “the Great Tumult,” was an insurrection that aimed to establish Munda Raj or Munda authority in this area.

Reason of revolt:

  • In 1874, the British replaced the traditional Khuntkari system of Munda tribals with the zamindari system.
  • The introduction of the zamindari system created the classes of zamindars (landlords) and ryots (tenants).
  • This led to the monetisation of the economy, the tribals had to depend on cash for paying the rent and for meeting their daily needs.
  • This situation left them with no other option but to depend on the money-lenders, who used to lend money at an exorbitant rate.
  • Birsa Munda:
    • A tribal liberation warrior from the Munda tribe, Bisra Munda is revered as a folk
    • He was a spearhead behind the Ulgulan movement that arose in the Chotanagpur region of Bihar and Jharkhand belt in the 19th century under the British
    • He is often referred to as the ‘Dharti Abba’ or the Earth Father.
    • Birsa Munda organised masses to stop paying debts/interest to moneylenders and taxes to the British. By this, he led a revolt to bring an end to Victorian rule and the establishment of Munda Rule in Jharkhand (Chotanagpur region).

     

    NOTE: If we go through Modern Ncert in detail, this question can be easily solved. It is advisable to remember keywords from Ncert such as Dikus, Ulgulan, Tebhaga, nij khamare dhan tolo etc. and corresponding movements.

 

Question 5

In the context of Indian history, the Rakhmabai case of 1884 revolved around:

  1. women’s right to gain education

  2. age of consent

  3. restitution of conjugal rights

Select the correct answer using the code given below:

(a) 1 and 2 only

(b) 2 and 3 only

(c) 1 and 3 only

(d) 1, 2 and 3

Ans: b

Sub-Theme: Social Reformist/Social Reform Organization

Dr. Rakhmabai (1864–1955)

  • Rakhmabai was an Indian physician and a feminist.
  • Apart from being one of the first practising women doctors in colonial India, she was also involved in a landmark legal case involving her marriage as a child bride between 1884 and 1888.
  • At the young age of eleven, she was married to Dadaji Bhikaji, who was then nineteen. Bhikaji petitioned the court in 1885, 12 years after they had been married, asking for the “restitution of conjugal rights.”
  • Behramji Malabari and Pandita Ramabai established the Rakhmabai Defense Committee to stand up for her.
  • The case lasted for 4 years until Dadji was compensated in 1888 out of court.
  • The case was instrumental in the development of the Age of Consent Act in stipulating minimum age of consent to be 12 years however no minimum age of marriage was introduced.
  • Statement 1 is incorrect: Dr. Rakhmabai was an Indian physician and a feminist. At the young age of eleven, she was married to Dadaji Bhikaji, who was then nineteen. Bhikaji petitioned the court in 1885, 12 years after they had been married, asking for the “restitution of conjugal rights.”The Rakhmabai case did not involve the women’s right to gain education.

  • Statement 2 is correct: The Rakhmabai case of 1884 was instrumental in the drafting of the Age of Consent Act in 1891. Hence, statement 2 is correct.

  • Statement 3 is correct: Dr. Rakhmabai was married off at the young age of eleven with Dadaji Bhikaji, who was nineteen years old. In 1885, after 12 years of marriage, Bhikaji filed a petition in the court and sought “restitution of conjugal rights”. Hence, statement 3 is correct.


 

Question 6

The Gandhi-Irwin Pact included which of the following?

1. Invitation to Congress to participate in the Round Table Conference

2. Withdrawal of Ordinances promulgated in connection with the Civil Disobedience Movement

3. Acceptance of Gandhi’s suggestion for enquiry into police excesses.

4. Release of only those prisoners who were not charged with violence.

Select the correct answer using the code given below:

(a) 1 only

(b) 1, 2 and 4 only

(c) 3 only

(d) 2, 3 and 4 only

Ans: b

Sub-Theme: Phases of Mahatma Gandhi/Gandhi- Irwin Pact

(UPSC didn’t give any answer in official key) Possible ans (b)

  • Statement 1 is correct: The Gandhi–Irwin Pact was a political agreement signed by Mahatma Gandhi and Lord Irwin, Viceroy of India, on 5 March 1931 before the Second Round Table Conference in London, where the Indian National Congress was invited to participate.
  • Statement 2 is correct: Withdrawal of all ordinances issued by the Government of India imposing curbs on the activities of the Indian National Congress was agreed by the Viceroy. Statement 3 is incorrect: The demand for a public enquiry of excesses committed by police during the civil disobedience movement was rejected by the Viceroy.
  • Statement 4 is correct: Release of all political prisoners, except those guilty of violence was also agreed.
  • NOTE: if we read Ncert, statement 1 is general and common. Securing participation by the Indian National Congress in the Second Round Table Conference was an important agenda of pact inter-alia. We also know that viceroy rejected commutation of the death sentence of Bhagat Singh, Rajguru and Sukhdev to a life sentence, which was criticised by several leaders. This underlines that prisoners with violence charges were not released. This reasoning makes statement 4 correct. Option (b) i.e 1, 2 and 4 only is the only option with statement 1 and 4 in it !!

 

Question 7

Wellesley established the Fort William College at Calcutta because:

(a) he was asked by the Board of Directors at London to do so

(b) he wanted to revive interest in oriental learning in India

(c) he wanted to provide William Carey and his associates with employment

(d) he wanted to train British civilians for administrative purpose in India

Ans: d

Sub-Theme: Reforms in Education during the British Time

Fort William College (1800):

•   Fort William College was founded on 10 July 1800 in Calcutta, British India.

  • Fort William College was set up by Lord Richard Wellesley in 1800
  • It was set up for the training of civil servants of the Company in languages and customs of Indians.
  • The court of directors of the British East India Company was not in support of a training college in Kolkata.
  • Then it got closed in 1802 and a separate College was established in 1807 in England.
NOTE: Question on Fort William College came in 2018. By simply reading solutions of that question, it was possible to attempt this question correctly, simply earning 2 marks !! This underscores the importance of solving and analysing PYQs.

 

Question 8

With reference to the book “Desher Katha” written by Sakharam Ganesh Deuskar during the freedom struggle, consider the following statement:

  1. It warned against the Colonial States hypnotic conquest of the mind.

  2. It inspired the performance of swadeshi street plays and folk songs.

  3. The use of ‘desh’ by Deuskar was in the specific context of the region of Bengal.

Which of the statements given above are correct?

(a) 1 and 2 only

(b) 2 and 3 only

(c) 1 and 3 only

(d) 1, 2 and 3

Ans: a

Sub-Theme: Literary Work during Indian National Movement

Statement 1 and 2 are correct: In overwhelming detail, the book “Desher Katha” summarised the work of M. G. Ranade and Naoroji in a popular idiom and warned in its concluding chapter against the colonial state’s “hypnotic conquest of the mind”. “Desher Katha” had an immense repercussion in Bengal, captured the mind of young Bengal and assisted more than anything else in the preparation of the Swadeshi Movement.

Statement 3 is incorrect: Deuskar uses the word ‘desh’ in the context of the whole country.


 

Question 9

The Vital-Vidhvansak, the first monthly journal to have the untouchable people as its target audience was published by:

(a) Gopal Baba Walangkar

(b) Jyotiba Phule

(c) Mohandas Karamchand Gandhi

(d) Bhimrao Ramji Ambedkar

Ans: a

Sub-Theme: Literary Work during Indian National Movement

Gopal Baba Walangkar (1840-1900):

  • He was also known as Gopal Krishna, born into a family of the untouchable Mahar caste in Maharashtra.
  • Gopal Baba Walangkar is an early example of a social activist working to untrammel the untouchable people of India from their historic socio-economic oppression and he is generally considered to be the pioneer of that movement.
  • He also developed a racial theory to explain the oppression and also published the first journal targeted at the untouchable people.
  • The first publication to have the untouchable people as its target readership was Walangkar’s Vital-Vidhvansak (Destroyer of Brahmanical or Ceremonial Pollution), which he started publishing in 1888.
  • He also wrote articles for Marathi-language newspapers such as Sudharak and Deenbandhu, as well as composed couplets in Marathi that were intended to inspire the people.
  • In 1889, he published Vital Viduvansan (Annihilation of Ceremonial Pollution), which protested the position of untouchables in society and raised consciousness regarding what those people should expect.
  • He addressed this pamphlet, which was crafted as a collection of 26 questions, to the elites of Maharashtrian society.


 

2019

 

Question 1

Which one of the following groups of plants was domesticated in the ‘New World’ and introduced into the ‘Old World’?

(a) Tobacco, cocoa and rubber

(b) Tobacco, cotton and rubber

(c) Cotton, coffee and sugarcane

(d) Rubber, coffee and wheat

Ans: a

Sub-Theme: Columbian Exchange

Tobacco, cocoa, and rubber are the groups of plants that were domesticated in the ‘New World’ and introduced into the ‘Old World’ through the European explorers in the late medieval or early modern era.

The Columbian Exchange:

  • The Columbian exchange, also known as the Columbian interchange, named for Christopher Columbus, was the widespread transfer of plants, animals, culture, human populations, technology, diseases, and ideas between the Americas, West Africa, and the Old World in the 15th and 16th centuries.
  • It also relates to European colonization and trade following Christopher Columbus’s 1492 voyage. Invasive species, including communicable diseases, were a by-product of the Exchange.
  • Old World to New World: Coffee, wheat, cotton, barley, rice, sugarcane and sugar beet, etc.
  • New World to Old World: avocado, cashew, cocoa bean, potato, maize, rubber, tobacco, etc.

NOTE: Even if we don’t know the exact answer of this particular question, we can still attempt this question with our basic knowledge. For instance, Cotton and wheat has been a widely cultivated crop in India since ancient times. Even Harappans grew

 

Question 2

Consider the following statements about ‘the Charter Act of 1813’:

  1. It ended the trade monopoly of the East India Company in India except for trade in tea and trade with China.

  2. It asserted the sovereignty of the British Crown over the Indian territories held by the company.

  3. The revenues of India were now controlled by the British Parliament.

Which of the statements given above are correct?

(a) 1 and 2 only

(b) 2 and 3 only

(c) 1 and 3 only

(d) 1, 2 and 3

Ans: a

Sub-Theme: Administrative Reforms before 1857

CHARTER ACT OF 1813:

  • Company’s territories in India came under the Sovereignty of the British Government.
  • Taxes could be levied against citizens by the local They could punish people for not paying taxes.
  • It ended the East India Company’s monopoly on trading with India.
  • The company’s rule was extended for another 20 years. The trade monopoly of their country was ended except for the trade in tea, opium, and with China.
  • Giving missionaries permission to travel to India was one of the key components of this law.

PITT’S INDIA ACT OF 1784:

  • Pitt’s India Act, 1784 extended the control of the British Government over the Company’s affairs and its administration in India.

The act was significant for two reasons:

  • The Company’s possessions in India were officially referred to as “British possessions in India” for the first time.
  • The Company’s activities and administration in India were placed under the total control of the British Government.
  • Statement 1 is correct: The Charter Act 1813 ended the trade monopoly of the East India Company except for the trade in tea, opium, and with China.
  • Statement 2 is correct: Company’s territories in India came under the Sovereignty of the British Government through this Act.
  • Statement 3 is incorrect: The revenue of India was controlled by the British parliament through Pitt’s India Act 1784.

 

Question 3

With reference to Swadeshi Movement consider the following statements:

  1. It contributed to the revival of the indigenous artisan crafts and industries.

  2. The National Council of Education was established as a part of the Swadeshi Movement.

Which of the statements given above is/are correct?

(a) 1 only

(b) 2 only

(c) Both 1 and 2

(d) Neither 1 nor 2

Ans: c

Sub-Theme: Swadeshi Movement

Swadeshi and Boycott Movements:

  • There were large-scale protests against the partition of Bengal. Later, the movement spread to other regions of the nation:
  • The Boycott Resolution was passed at the Calcutta Town Hall on August 7, 1905, formally launching the Swadeshi Movement.
  • Partition came into force: October 16, 1905
  • There were also new means of protest like processions, public meetings, and boycotts of imported goods.
  • Imaginative use of Traditional Festivals, Melas: Tilak’s Ganapati and Shivaji festivals became a medium of swadeshi
  • Statement 1 is correct: Importance to Self- Reliance (Atma Shakti): Emphasis was placed on honour, social and economic regeneration of the villages. Swadeshi enterprises: Swadeshi textile mills, banks, etc, were set up. At Tuticorin, V.O. Chidambaram Pillai founded the Swadeshi Steam Navigation Company.
  • Statement 2 is correct: Swadeshi programme or National Education:
  1. Bengal National College – Aurobindo Ghosh as its Principal – inspired by Tagore’s
  2. The National Council of Education (1906) was set up to organise national education in the vernacular medium.
  3. The Bengal Institute of Technology was established to provide technical training.
    • Cultural Impact: Rabindranath Tagore wrote Amar Sonar Bangla, Subramania Bharati wrote Swadesha Geetham.
    • Corps of Volunteers or ‘Samitis’: they generated political consciousness among the masses.

NOTE: Question on similar theme/topic i.e Surat Split/Partition of Bengal/Swadeshi Movement came in 2014 (on annulment of partition of Bengal), 2015 and 2016 (total 2 questions). Hereby, revising PYQ stands important.

 

Question 4

With reference to the British colonial rule in India, consider the following statements:

  1. Mahatma Gandhi was instrumental in the abolition of the system of ‘indentured labour’.

  2. In Lord Chelmsford’s War Conference’, Mahatma Gandhi did not support the resolution on recruiting Indians for World War.

  3. Consequent upon the breaking of the Salt Law by the Indian people, the Indian National Congress was declared illegal by the colonial rulers.

Which of the statements given above are correct?

(a) 1 and 2 only

(b) 1 and 3 only

(c) 2 and 3 only

(d) 1, 2 and 3

Ans: b        

Statement 1 is correct: During the early 1900s, Mahatma Gandhi was instrumental in getting the indentured labour system abolished in the British Empire. He also supported Manilal Doctor’s campaign to get a resolution passed in the INC meeting condemning the indentured labour system in all British territories.

Statement 2 is incorrect: Lord Chelmsford, the then Viceroy of India, invited Gandhi to Delhi at a War Conference. In order to gain the trust of the empire, Gandhi agreed to move people to enlist in the army for WW-I.

Statement 3 is correct: Indian National Congress was declared illegal upon breaking of salt law. But this did not deter the satyagrahis who continued the movement.

 

Question 5

With reference to Indian National Movement, consider the following pairs:

Person Position held
1. Sir Tej Bahadur Sapru President, All India Liberal Federation
2. K. C. Neogy Member, The Constituent Assembly
3. P. C. Joshi General Secretary, Communist Party of India

Which of the pairs given above is/are correctly matched?

(a) 1 only

(b) 1 and 2 only

(c) 3 only

(d) 1, 2 and 3

Ans: d        

Sub-Theme: Important Personalities/ Important Organisations

Pair 1 is correct: All India Liberal Federation was founded by Surendra Nath Banarjea and some of its prominent leaders were Tej Bahadur Sapru, V. S. Srinivasa Sastri and R. Jayakar and Tej Bahadur Sapru were the presidents of Indian National Liberal Federation.

Pair 2 is correct: K.C. Neogy was a member of the Constituent Assembly of India, a member of the first Cabinet of independent India and the chairman of the first Finance Commission of India.

Pair 3 is correct: P.C. Joshi was one of the early leaders of the communist movement in India, he was the first general secretary of the Communist Party of India from 1935–47.


 

Question 6

Consider the following pairs:

Movement Organization Leader
1. All India Anti- Untouchability League Mahatma Gandhi
2. All India Kisan Sabha Swami Sahajanand Saraswati
3. Self-Respect Movement E. V. Ramaswami Naicker

Which of the pairs given above is/are correctly matched?

(a) 1 only

(b) 1 and 2 only

(c) 2 and 3 only

(d) 1, 2 and 3

Ans: d

Sub-Theme: Important Organisations

Pair 1 is correct: On 30 September 1932, Mahatma Gandhi founded the All India Anti Untouchability League, to remove untouchability in society.

Pair 2 is correct: Sahajanand Saraswati was the first President of the All-India Kisan Sabha.

Pair 3 is correct: The Self-Respect Movement was started by E.V. Ramaswamy Naicker in Tamil Nadu in 1925 with the aim of destroying the contemporary Hindu social order in its totality and creating a new, rational society without caste, religion and god.

 

2018

 

Question 1

Which one of the following statements does not apply to the system of Subsidiary Alliance introduced by Lord Wellesley?

(a) To maintain a large standing army at others expense

(b) To keep India safe from Napoleonic danger

(c) To secure a fixed income for the Company

(d) To establish British paramountcy over the Indian States

Ans: c

Sub-Theme: British Expansionist Policy

The system aimed to establish British paramountcy over the Indian States and to ensure their military and political subservience to the British East India Company. Securing a fixed income for the company was not the primary objective of the system.

Subsidiary Alliance by Wellesley (1798- 1805):

  • Allying Indian states maintained the British army at its frontier and paid for their maintenance. Ruler had to post British officials at their Ruler couldn’t employ Europeans without the British’s permission. Couldn’t contact enemies to make peace.
  • Maintenance asked was high → Ruler’s couldn’t pay → asked to cede part of their territory.
  • Awadh was first to come under it.
  • Signed by: Nawab of Awadh, Nizam of Hyderabad, Ruler of Mysore, Ruler of Tanjore, Peshwa, Bhonsle of Berar, The Sindhias, The Rajput states, The ruler of Bharatpur, Holkars.
  • During this phase, almost all the Indian States came under British subordination, by compelling them to enter into subsidiary alliances with Also the Indian Princes were put under obligation to accept the British Paramountcy.
  • It was a ‘Non-Intervention Policy’ used by Lord Wellesley to bring the Princely states under the control of the British.
  • Though the native Indian ruler had to pay for the maintenance of the British army and if he failed to make the payment, a portion of his territory was taken away and ceded to the British, but securing fixed income was never part of the system of Subsidiary alliance.

NOTE: Always stay focused and calm while solving this type of question, often in this type of question we miss/overlook the negative keyword such as ‘Does Not’/’Not’/’Not Correct’, etc. in the question and mark the wrong answer in a haste.

 

Question 2

Economically, one of the results of the British rule in India in the 19th century was the:

(a) increase in the export of Indian handicrafts

(b) growth in the number of Indian owned factories

(c) commercialization of Indian agriculture

(d) rapid increase in the urban population

Ans: c

Sub-Theme: Commercialisation of Indian Agriculture

One of the results of the British rule in India in the 19th century was the commercialization of agriculture by introducing cash crops like opium and indigo

  • Colonial rule has negatively impacted the Indian industries and handicrafts.
  • The increase in the import of finished manufactured goods poses a threat to the indigenous factories.
  • It was, economically, one of the results of British rule in India in the 19th Century was the growth of commercial crops like cotton, jute, groundnut, oilseeds, tobacco, sugarcane, etc. and the raw materials for the industries in Britain.
  • There was no rapid increase in the urban population during 19th Century British India.

NOTE: If we read NCERT/Spectrum carefully, we can easily observe a pattern in given sentences: in sentence 1, “decrease” replaced by “increase”, in sentence 2, “foreign” replaced by “Indian” and in sentence 3, “rural” is replaced by “urban”.

 

Question 3

The staple commodities of export by the English East Indian Company from Bengal in the middle of the 18th century were

(a) Raw cotton, oil-seeds and opium

(b) Sugar, salt, zinc and lead

(c) Copper, silver, gold, spices and tea

(d) Cotton, silk, saltpeter and opium

Ans: d

Sub-Theme: Colonial Trade

The East India Company was originally formed in Britain for pursuing trade with the East Indies in Southeast Asia. In fact, it ended up trading mainly with the Indian subcontinent and China, where the main items of trade were cotton, silk, tea, opium, and saltpetre (potassium nitrate).

 

Question 4

In the Federation established by the Government of India Act of 1935, residuary powers were given to the:

(a) Federal Legislature

(b) Governor General

(c) Provincial Legislature

(d) Provincial Governors

Ans: b

Sub-Theme: Administrative Reforms post 1857 As per the Government of India Act of 1935 Governor-General was empowered to authorise, either the Federal or the Provincial Legislature to enact a law with respect to any residuary matter.

The Government of India Act, 1935

  • Divided the powers between the Centre and provinces in terms of three lists– Federal List (for Centre, with 59 items), Provincial List (for provinces, with 54 items) and the Concurrent List (for both, with 36 items).
  • The allocation of residuary powers was unique. It was not vested in either of the legislatures, central or But the Governor-General was empowered to authorise, either the Federal or the Provincial Legislature to enact a law with respect to any residuary matter.
  • Abolished dyarchy in the provinces and introduced ‘provincial autonomy’ in its place, while adopted the dyarchy at the Centre
  • Enacted bicameralism in six out of eleven provinces
  • It further extended the principle of communal representation to the depressed classes (Scheduled Castes), women and labour (workers).
  • Abolished the Council of India, established by the Government of India Act of 1858.

NOTE: Questions from this particular theme were generally straightforward in nature, also this particular theme is a very high yielding theme both in terms of Modern History and Polity, so read thoroughly as you cannot afford to miss these low hanging fruits.


 

Question 5

After the Santhal Uprising subsided, what was/ were the measure/measures taken by the colonial government?

  1. The territories called ‘Santhal Paraganas’ were created.

  2. It became illegal for a Santhal to transfer land to a non-Santhal.

Select the correct answer using the code given below:

(a) 1 only

(b) 2 only

(c) Both 1 and 2

(d) Neither 1 nor 2

Ans: c

Sub-Theme: Tribal Movement

Both the given measures were taken by the Colonial Government as a result of Santhal Uprising.

Santhal Uprising:

  • Santhals: They used to live in the Daman- i-Koh area which is between Bhagalpur and Rajmahal, and ‘hool’ means uprising in the Santhali language.
  • The Santhals of the Rajmahal Hills were resentful of the oppression they suffered at the hands of revenue officials, police, money- lenders, and landlords, by the “outsiders” (whom they called diku).
  • In 1854, the Santhals under the leadership of Sido and Kanhu rebelled against their oppressors, declared the end of the Company’s control, and proclaimed their independence.
  • The situation wasn’t brought under control until 1856, a year that saw extensive military Sido died in 1855, while Kanhu was arrested in 1866.
  • A separate district of Santhal Parganas was created by the Government to pacify the Santhals. Thus, the government separated Santhal Parganas into its own district in order to pacify the Santhals.
  • It became illegal for a Santhal to transfer land to a non-Santhal. British Government enacted laws so that the lands of tribals could not be taken by outsiders (dikus).


 

Question 6

Which one of the following is a very significant aspect of the Champaran Satyagraha?

(a) Active all-India participation of lawyers, students and women in the National Movement

(b) Active involvement of Dalit and Tribal communities of India in the National Movement

(c) Joining of peasant unrest to India’s National Movement

(d) Drastic decrease in the cultivation of plantation crops and commercial crops

Ans: c

Sub-Theme: Phase of Mahatma Gandhi Champaran Satyagraha (1917):

  • Mahatma Gandhi’s initial experiment of satyagraha was known as Champaran Satyagraha.
  • It was undertaken after Mahatma Gandhi learned about the abuses suffered by farmers, who were forced into growing indigo by British planters and estate
  • The tenants from Champaran were forced under the law to grow indigo on 3/20th part of the land (Tinkathia System) for his landlord.
  • To increase their profits, European planters demanded exorbitant rents and illegitimate
  • Rajkumar Shukla invited Gandhi to look into the problems of Indigo planters in Champaran, Bihar.
  • Gandhiji intended to conduct a thorough investigation in the district and demand action based on its results.
  • During this time Gandhiji was even charged with violating law and was told to leave Champaran, but he refused to leave.
  • Later, the then Lieutenant Governor of Bihar ordered the withdrawal of the case against Gandhi, and the Collector wrote to Gandhi saying he was free to conduct the inquiry.
  • In the history of the independence struggle, this tiny step in the form of passive protest was a great leap, signalling the beginning of the Gandhian period.
  • Leaders associated: Rajendra Prasad, Mazhar-ul-Haq, Mahadeo Desai, Narhari Parekh, J.B. Kripalani etc.
  • Gandhi’s position in India’s resistance to the British raj was solidified by the victory at Champaran.

NOTE: in given statements, notice “allIndia participationin statement (a) and “Drastic decreasein statement (d), we can take risk of eliminating these extreme statements. Always remember, there is no set rule for this “extreme word” elimination method. There are also some instances where sentences with extreme words were correct, for instance a question in 2017 about Foreign exchange of India. In the mentioned question, the statement with “drastic increase” was correct.

 

Question 7

In 1920, which of the following changed its name to “Swarajya Sabha”?

(a) All India Home Rule League

(b) Hindu Mahasabha

(c) South Indian Liberal Federation

(d) The Servants of India Society

Ans: a

Sub-Theme: Home Rule League Movement

Home Rule League Movement (1916):

  • It was an Indian response to World War – I, inspired by Irish Home Rule Leagues.
  • The movement sought to achieve self-rule through political discussion and education, public gatherings, etc.
  • The government responded with severe repression, but it prepared masses for Gandhian style of politics.
  • Tilak launched the Indian Home Rule League in April 1916 at Belgaum.
  • Annie Besant launched the Home Rule League in September 1916 at Madras.
  • The All India Home Rule League became Swarajya Sabha in 1920.
  • Its demands included Swarajya, formation of linguistic states and education in the vernacular.
  • Leaders: Motilal Nehru, Lala Lajpat Rai, Jawaharlal Nehru, Bhulabhai Desai, Chittaranjan Das, Madan Mohan Malaviya, Mohammad Ali Jinnah, Tej Bahadur Sapru.
  • Anglo-Indians, most of the Muslims and Non brahmins from south did not join as they felt it would mean home rule of the Hindu majority.


 

Question 8

Regarding Wood’s Dispatch, which of the following statements are true?

  1. Grants-in-Aid system was introduced.

  2. Establishment of universities was recommended.

  3. English as a medium of instruction at all levels of education was recommended.

Select the correct answer using the code given below:

(a) 1 and 2 only

(b) 2 and 3 only

(c) 1 and 3 only

(d) 1, 2 and 3

Ans: a

Sub-Theme: Reforms in Education during the British Time

Wood’s Dispatch (1854):

  • Magna Carta of English Education in India.
  • Government to take up the responsibility for the Education of masses.
  • Hierarchy of Educational Institutions
  • Emphasise on       Female,       Vocational Education and teachers’ training.
  • Secular Education in Government Institutions. 
  • Grants-in-aid to encourage private education.
  • Recommended English as the Medium of Instruction in higher studies and vernaculars in school level.
  • Universities with affiliations in three presidency towns of Calcutta, Bombay, and Madras.

Statement 1 and 2 are correct: Grants-in-aid to encourage private education and affiliating universities in the presidency towns of Calcutta, Bombay and Madras were recommended in the Wood’s Despatch.

Statement 3 is incorrect: The Woods Dispatch recommended English as the Medium of Instruction in higher studies and vernaculars in school level.

NOTE: Grants-in-Aid system and Establishment of universities was very popular recommendations of Woods Dispatch, so option 1 and 2 are correct. However, if you look carefully at option 3, it talks about English as a medium of instruction at all levels of education. Dispatch was in 1854, at that time introducing English as a medium of instruction at all levels sounds unconvincing. By applying this crude logic, and taking the risk of eliminating option 3, we can get the answer.

 

Question 9

Which of the following led to the introduction of English Education in India?

  1. Charter Act of 1813

  2. General Committee of Public Instruction, 1823

  3. Orientalist and Anglicist controversy.

Select the correct answer using the code given below:

(a) 1 and 2 only

(b) 2 only

(c) 1 and 3 only

(d) 1, 2 and 3

Ans: d

Sub-Theme: Reforms in Education during the British Time

Statement 1 is correct: The Charter Act of 1813 permitted Christian missionaries to propagate English and preach their religion.

Statement 2 is correct: As a president of the General Committee of Public Instruction, Lord Macaulay wrote a minute, where he Macaulay stressed the implementation of the English language as a medium of instruction through his minute.

Statement 3 is correct: One faction of the Anglicists was in favour of English language as the medium, while the other faction was in favour of Indian languages (vernaculars).

Charter Act of 1813:

  • The idea of supporting educated Indians and promoting knowledge of contemporary sciences in the country was enshrined into the Charter Act of 1813.
  • The Act required the Company to yearly approve one lakh rupees for this Even this meagre sum, though, was not made accessible until 1823, mostly due to the disagreement over the most appropriate strategy for this investment.
  • The Charter Act of 1813 gave Christian missionaries the right to spread the English language and practise their religion.

General Committee on Public Instruction (1823):

  • A “General Committee of Public Instruction” was established by the Governor-General-in-Council in 1823 with the task of allocating one lakh rupees for educational purposes.
  • The General Committee of Public Instruction also was not able to decide the medium of instruction by vote; due to lack of majority.
  • Out of ten members of which Lord Macaulay was the president, five were supporters of English language or Anglicist as the medium of instruction and the rest were supporters of oriental or classic language or Classicists as a medium of instruction.
  • This is the famous Anglicist and Classicist controversy.
  • On February 2, 1835, Lord Macaulay, who served as head of the General Committee of Public Instruction, penned a minute in which he stated his position on the issue.
  • Through his minute, Lord Macaulay emphasised the use of the English language as a medium of instruction. He believed that English was the ideal language for instruction.

Orientalist-Anglicist controversy:

•   Exclusively suggested for spending on modern studies.
 

 

Anglicists opinion

•   Even the Anglicists were divided over the question of medium of instruction, where one faction was for English language as the medium, while the other faction was for Indian languages (vernaculars) for the purpose.
 

 

Orientalists

•   In order to prepare students for careers, Western sciences and literature should be taught, with a focus on advancing traditional Indian education.

 

Question 10

With reference to educational institutions during colonial rule in India, consider the following pairs: Institution Founder:

  1. Sanskrit College at Benaras – William Jones

  2. Calcutta Madrasa – Warren Hastings

  3. Fort William College – Arthur Wellesley

Which of the pairs given above is/are correct?

(a) 1 and 2

(b) 2 only

(c) 1 and 3

(d) 3 only

Ans: b       

Sub-Theme: Reforms in Education during the British Time

Statement 1 is incorrect: The Sanskrit College was established by Jonathan Duncan, the resident, at Benaras in 1791 for study of Hindu law and philosophy.

Statement 2 is correct: The Calcutta Madrasah was established by Warren Hastings in 1781 for the study of Muslim law and related subjects.

Statement 3 is incorrect: Fort William College was set up by Lord Richard Wellesley in 1800 for training of civil servants of the Company in languages and customs of Indians (closed in 1802).

 

Question 11

He wrote biographies of Mazzini, Garibaldi, Shivaji and Shri krishna; stayed in America for some time; and was also elected to the Central Assembly. He was:

(a) Aurobindo Ghosh

(b) Bipin Chandra Pal

(c) Lala Lajpat Rai

(d) Motilal Nehru

Ans: c

Sub-Theme: Important Personalities

Lala Lajpat Rai:

  • Popularly known as Punjab Kesari. A member of the Lal Bal Pal trio.
  • The INC’s president at the 1920 Calcutta Special Session.
  • In 1921, he founded Servants of the People Society, a non-profit welfare organisation, in Lahore.
  • Lajpat Rai travelled to the US in 1917.
  • Led a non-violent march in protest to the Simon Commission. Brutally assaulted by the police and later succumbed to his
  • Along with founding Arya Gazette as its editor, authored:
    1. The Story of My Deportation, 1908.
    2. Arya Samaj, 1915.
    3. The United States of America: A Hindu’s Impression, 1916.
    4. The problem of National Education in India, 1920
    5. Unhappy India, 1928.
    6. England’s Debt to India, 1917.
    7. Autobiographical Writings
    8. He also wrote biographies of Mazzini, Garibaldi, Shivaji, and Shrikrishna.

 

Question 12 

Which among the following events happened earliest?

(a) Swami Dayanand established Arya Samaj.

(b) Dinabandhu Mitra wrote Neeldarpan.

(c) Bankim Chandra Chattopadhyay wrote Anandmath.

(d) Satyendranath Tagore became the first Indian to succeed in the Indian Civil Services Examination

Ans: b

Sub-Theme: Literary Work during Indian National Movement

Option (a) is incorrect: The Arya samaj was founded by the Dayananda Saraswati on 7 April 1875.

Option (b) is correct: Neel Darpan is a Bengali play, written by Dinabandhu Mitra in 1858–1859.

Option (c) is incorrect: Anandamath the famous Bengali novel written by Bankim Chandra Chatterjee in 1882.

Option (d) is incorrect: Satyendranath Tagore was selected for Indian Civil Service in June 1863. He was the first Indian to succeed in the ICS Exam in 1863.

NOTE: As we have mentioned earlier, a chronology of events is important. This is another question on the same theme, earlier questions were asked in 2017 and 2021.

 

2017

 

Question 1

Who among the following was/were associated with the introduction of Ryotwari Settlement in India during the British Rule?

  1. Lord Cornwallis

  2. Alexander Read

  3. Thomas Munro

Select the correct answer using the code given below:

(a) 1 only

(b) 1 and 3 only

(c) 2 and 3 only

(d) 1, 2 and 3

Ans: c

Sub-Theme: Land Revenue Settlement

Permanent Settlement
  • Started by Lord Cornwallis in 1793.
  • Prevalent in Bengal, Bihar, Orissa, Varanasi (United Provinces), and Northern Karnataka.
  • Covered 19% of British India.
  • A new section of Zamindars was appointed who would provide 10/11th part of the revenue collected to the British while keeping the remaining.
 

 

Ryotwari System

  • Started by Thomas Munro and Alexander Reed in 1820.
  • Started in the Baramahal district of Tamil Nadu and later spread to Madras, parts of Bombay, East Bengal, Assam and Kurg (Karnataka).
  • Covered 51% of British India.
  • Ryots (farmers) were given ownership and other rights over the land and were required to pay the revenue directly to the government. Involved high taxes and strict collection.
 

 

Mahalwari System

  • The Mahalwari system was introduced by Holt Mackenzie in 1822.
  • Prevalent in United Provinces, Central Provinces and Punjab.
  • Covered 30% of British India.
  • The village or mahal served as the unit for collecting revenue. Village land belonged to the community and it was the responsibility of the entire community to pay the land revenue.

 

NOTE: Questions on the settlement system earlier came in 2011 and two questions in 2012. Reference of PYQs and research about given options stands important.

 

Question 2. The Trade Disputes Act of 1929 provided for:

(a) the participation of workers in the management of industries.

(b) arbitrary powers to the management to quell industrial disputes.

(c) an intervention by the British Court in the event of a trade dispute.

(d) a system of tribunals and a ban on strikes.

Ans: d

Sub-Theme: Labour Legislations

About the Trade Disputes Act, 1929:

  • The Trade Disputes Act, 1929 was codified for five years as an experimental measure.
  • The main objective of the Act was to make provisions for the establishment of Courts of Inquiry and Boards of Conciliation with a view to investigate and settle trade disputes.
  • The Act banned unnoticed strikes or lockouts in the provision of public utility services.
  • A strike or lockout that was intended to cause severe, widespread, or prolonged hardship for the community and compel the government to take or refrain from taking a particular course of action was also made illegal.
  • This included strikes and lockouts that had any purpose other than to advance a trade dispute within the trade or industry in which the strikers or the employers locking out were engaged.
  • The Act was amended in 1932 and was made permanent by the Trade Disputes (Extending) Act, 1934.


 

Question 3. Consider the following statements:

  1. The Factories Act, 1881 was passed with a view to fix the wages of industrial workers and to allow the workers to form trade unions.

  2. M. Lokhande was a pioneer in organizing the labour movement in British India.

Which of the above statements is/are correct?

(a) 1 only

(b) 2 only

(c) Both 1 and 2

(d) Neither 1 nor 2

Ans: b

Sub-Theme: Labour Legislations/Labour Movement

Statement 1 is incorrect: The first Factory Act (1881) was passed in 1881 during the tenure of Lord Ripon. This legislation aimed to improve working conditions for factory workers. This act applies only to factories that use mechanical power, employing at least 100 workers. It prohibited employment of children under the age 7. Children between the ages 7-12 were to work for a maximum 9 hours. It also made compulsory that dangerous machinery should be fenced properly, but no mention of forming trade unions.

Statement 2 is correct: Narayan Meghaji Lokhande (1848–1897) was the father of the trade union movement in India. He is remembered not only for ameliorating the working conditions of textile mill-hands in the 19th century but also for his courageous initiatives on caste and communal issues. Lokhande is acclaimed as the Father of the Trade Union Movement in India. Some of the rights mill workers got because of N M Lokhande were:

  • Mill workers should get a weekly holiday on Sunday
  • In the afternoon, workers should be entitled to half-hour recess.
  • The mill should start working from 6:30 in the morning and close by sunset.
  • The salaries of the workers should be given by the 15th of every month.
NOTE: From NCERT, we know that N.M. Lokhande was a pioneer of the labour movement in India and widely regarded as the “father of the trade union movement”. He also founded “Mill Hands’ Association”. If we critically think about statement 1, it states that Factories Act, 1881 allows the workers to form trade unions. It was the very first Factories Act, 1881. Do you think Britishers would give such a heavy right in its very first act rather than incremental moves? This reasoning makes the possibility of statement 1 being incorrect.

Read carefully about various personalities, at least about those which are mentioned in Modern NCERT. Personalities from Swadeshi Movement, Non-Cooperation Movement, CDM, Quit India Movement (esp. Underground activities) and various socio- religious reform movements are important.

 

Question 4

In the context of Indian history, the principle of ‘dyarchy (diarchy)’ refers to:

(a) division of the central Legislature into two houses.

(b) introductions of double Government e., central and Statement governments.

(c) having two sets of rulers; One in London and another in Delhi.

(d) division of the subjects delegated to the provinces into two categories.

Ans: d

Sub-Theme: Administrative Reforms post 1857/ Montagu-Chelmsford Reforms/GoI Act 1919

Government of India Act/Montagu-Chelmsford Act, 1919 divided the provincial subjects into two parts: transferred and reserved, this dual scheme of governance was known as ‘dyarchy’.

Introduction to Diarchy:

  • The GoI Act 1919 introduced a diarchy (rule of two individuals/parties) for the executive at the level of the provincial government.
  • The diarchy was implemented in eight provinces: Assam, Bengal, Bihar and Orissa, Central Provinces, United Provinces, Bombay, Madras and Punjab.
  • The Dyarchy system increased the authority of the provincial governments.
  • In the province, the governor was to serve as the chief executive.

Division of Subjects:

  • Division of subjects into two lists: ‘reserved’ and ‘transferred’.
  • The reserved list: included subjects such as law and order, finance, land revenue, irrigation and the subjects were to be administered by the governor through his executive council of bureaucrats. All significant subjects were maintained in the Provincial Executive’s reserved subjects.
  • The transferred subjects: were to be administered by ministers nominated from among the elected members of the legislative council and it included subjects such as education, health, local government, industry, agriculture, excise, etc.

NOTE: UPSC has asked several questions from this particular theme/sub-theme, thus aspirants are requested to read this particular theme very minutely both from Spectrum and Laxmikanth.

 

Question 5

The object of the Butler Committee of 1927 was to:

(a) Define the jurisdiction of the Central and Provincial Governments.

(b) Define the powers of the Secretary of State for India.

(c) Impose censorship on national Press.

(d) Improve the relationship between the Government of India and the Indian States.

Ans: d

Sub-Theme: Important Commission during British

Butler Committee (1927):

  • The Butler Committee was appointed under the chairmanship of Sir Harcourt Butler in 1927 to investigate and clarify the relationship between the paramount power of the British Raj in India, and the rulers of Princely States.
  • There were two other members of the committee: William Searle Holdsworth and Sidney Peel.

Recommendations:

  • In the committee’s report of 1929, the “paramountcy” doctrine was reaffirmed.
  • Guidelines were given for its application, and it was made clear that the financial relationship between the Raj and the States should be fair.
  • The relationship of the princely states with the British empire was not merely a contractual relationship, but a living, growing relationship shaped by the circumstances and policy, resting on the mixture of history and theory.
  • British paramountcy to stay intact(solid) to preserve the princely state.
  • State should not be transferred without their own consent to a relationship with a new government in British India responsible to an Indian legislature.

 

Question 6

With reference to Indian freedom struggle, consider the following events:

  1. Mutiny in Royal Indian Navy

  2. Quit India Movement launched

  3. Second Round Table Conference

What is the correct chronological sequence of the above events?

(a) 1-2-3

(b) 2-1-3

(c) 3-2-1

(d) 3-1-2

Ans: c

Sub-Theme: Last phase of Indian Freedom Struggle/RIN Movement

Second Round Table Conference (1931):

  • In London, between September 7, 1931 to December 1, 1931.
  • Result of the Gandhi-Irwin
  • Gandhi was the sole representative of the
  • Madan Mohan Malviya and A. Rangaswami Iyengar were also there.
  • Muslim league represented by Aga Khan III, Maulana Mohammad Jinnah.
  • Depressed classes by B. R. Ambedkar, Rettamalai Srinivasan, Sardar Sampuran Singh etc.
  • Sarojini Naidu participated to represent
  • Lord Willingdon was the viceroy of
  • A Communal Award for representing minorities in India by providing for separate electorates for minority communities. Gandhi was against this.
  • The government refused Indian demands and talks failed.

Quit India Movement/August Revolution (1942):

  • After the failure of Cripps’s Mission, a Resolution was framed by Gandhiji for British withdrawal and  non-violent Non-Cooperation Movement against the Japanese Invasion.
  • The resolution was accepted in the CWC meeting in Wardha on July 14, 1942.

Royal Indian Naval Mutiny (1946):

  • On February 18, 1946, a mutiny started among the ratings of HMIS Talwar due to poor pay, inadequate food, and racial
  • The Muslim League, Congress, and CPI’s red flags were raised.
  • The intervention of Sardar Patel put an end to the rebellion. The mutineers faced trials despite promises.
  • This led to Violence in Bombay, with the loss of 200 lives.
  • Significance: On February 19, 1946, a Cabinet Mission was sent to India.
NOTE: this question underscores the importance of understanding chain of events and remembering relative timeline. In 2021, UPSC asked what happened on the 8th Aug 1942. So it is advisable to remember the chronology of important events.

 

Question 7

Consider the following pairs:

  1. Radhakanta Deb – first President of the British Indian Association

  2. Gazulu Lakshminarasu Chetty – founder of the Madras Mahajan Sabha

  3. Surendranath Banerjee – Founder of the Indian Association

Which of the above pairs is/are correctly matched?

(a) 1 only

(b) 1 and 3 only

(c) 2 and 3 only

(d) 1, 2 and 3

Ans: b        

Sub-Theme: Social Movement Organisations

  • Pair 1 is correct: Raja Radhakanta Deb was the first President of the British Indian Association while Debendranath Tagore was its secretary.
  • Pair 2 is incorrect: In 1884 Madras Mahajan Sabha  was established by M. Veeraraghavachariar, G. Subramania Iyer and P. Anandacharlu.
  • Pair 3 is correct: Surendranath Banerjee and Anand  Mohan  Bose  founded  the Indian Association of Calcutta in 1876.

 

2016

 

Question 1

The Montague-Chelmsford Proposals were related to:

(a) social reforms

(b) educational reforms

(c) reforms in police administration

(d) constitutional reforms

Ans: d

Sub-Theme: Administrative Reforms/Montague- Chelmsford Reforms Act

Montague-Chelmsford Proposals had the scheme of constitutional reforms, known as the Montagu-Chelmsford (or Mont-Ford) Reforms, which led to the enactment of the Government of India Act of 1919.

  • The GoI Act 1919 introduced a diarchy (rule of two individuals/parties) for the executive at the level of the provincial government.
  • The diarchy was implemented in eight provinces: Assam, Bengal, Bihar and Orissa, Central Provinces, United Provinces, Bombay, Madras and Punjab.
  • The Dyarchy system increased the authority of the provincial governments.
  • In the province, the governor was to serve as the chief executive.

 

Question 2

What was the main reason for the split in the Indian National Congress at Surat in 1907?

(a) Introduction of communalism into Indian politics by Lord Minto

(b) Extremists’ lack of faith in the capacity of the moderates to negotiate with the British Government

(c) Foundation of Muslim League

(d) Aurobindo Ghosh’s inability to the elected as the President of the Indian National Congress

Ans: b

Sub-Theme: Important INC Sessions/Surat Split/ Swadeshi Movement

Option (a) is incorrect: Morley-Minto Reforms, 1909 introduced separate electorates  on  the  basis  of  religion, thus introducing a system of communal representation. This was not the reason for the Congress split in 1907.

Option (b) is correct: On the backdrop of the Swadeshi Movement, the Extremists wanted either B.G. Tilak or Lajpat Rai as President. Moderates proposed Rashbehari Ghosh and wanted the session to be in Surat in order to exclude Tilak from the presidency. Since a leader from the host province could not be session president, they also sought to drop resolutions on swadeshi, boycott and national education. Thus it resulted in a split in the congress. Therefore, Extremists’ lack of faith in the capacity of the moderates to negotiate with the British Government’.

Option (c) is incorrect: Formation of Muslim League in 1906 at Dhaka was not the reason for Congress split in 1907.

Option (d) is incorrect: This is irrelevant to the particular question.

NOTE: Question on similar theme/topic i.e Surat Split/Swadeshi Movement came in 2015.

 

Question 3

The ‘Swadeshi’ and ‘Boycott’ were adopted as methods of struggle for the first time during the:

(a) agitation against the Partition of Bengal

(b) Home Rule Movement

(c) Non-Cooperation Movement

(d) visit of the Simon Commission to India

Ans: a

Sub-Theme: Swadeshi Movement

Option (a) is correct: The ‘Swadeshi’ and ‘Boycott’ were the mass movements to oppose the Bengal partition.

Option (b) is incorrect: The Home Rule Movement in 1916 was the Indian response to the First World War in a less charged but in more effective way. With people already feeling the burden of war time miseries caused by high taxation and a rise in prices, Tilak and Annie Besant ready to assume the leadership, the movement started with great vigour.

Option (c) is incorrect: The time period of Swadeshi and Boycott Movement and Non- Cooperation Movement is different. Therefore it has no correlation with the Swadeshi and Boycott Movement.

Option (d) is incorrect: Simon Commission visited in 1928 to review the constitutional reform in Britain’s largest and most important possession.


 

Question 4

The plan of Sir Stafford Cripps envisaged that after the Second World War:

(a) India should be granted complete independence

(b) India should be partitioned into two before granting independence

(c) India should be made a republic with the condition that she will join the Commonwealth

(d) India should be given Dominion status

Ans: d

Sub-Theme: Cripps Mission

Cripps Mission (1942):

  • In March 1942, a mission headed by Stafford Cripps was sent to India with constitutional proposals to seek Indian support for World War II.
  • Proposals:
    • Indian union with a Dominion status would be set up and it would be free to decide its relation with the UN and the Commonwealth.
    • After the war, the Constituent Assembly would be formed with members from Provinces (elected through proportional representation) and Princely states (nominated).
  • Conditions of accepting the New Constitution:
    •  Any Province not willing to join the Union can have a separate Constitution and form a separate union.
    •  New Constitution-making Body and the Government would negotiate a treaty for Transfer of Power.

 

Question 5

Satya Shodhak Samaj organized:

(a) a movement for upliftment of tribals in Bihar

(b) a temple-entry movement in Gujarat

(c) an anti-caste movement in Maharashtra

(d) a peasant movement in Punjab

Ans: c

Sub-Theme: Social Reform Organization

An anti-caste movement was organized by Satya Shodhak Samaj in Maharashtra.

Satyashodhak Samaj:

  • On 24 September 1873, Jotirao Govindrao Phule formed Satyashodhak Samaj to focus on the rights of depressed groups such as women, the Shudra, and the Dalit.
  • The Satya Shodhak Samaj opposed idolatry and condemned the caste system by doing
  • The Satyashodhak Samaj opposed the need for priests and advocated for the spread of reason.
  • Jotirao Govindrao Phule founded Satyashodhak Samaj with the goals of promoting the happiness, well-being, equality, and simplicity of religious practices for all people.
  • The Deenbandhu newspaper, published from Pune, gave the Samaj’s viewpoints a
  • Muslims, Brahmins, and government officials all belonged to the Leading members and financial backers for the group came from Phule’s own Mali caste.

 

Question 6

Consider the following:

  1. Calcutta Unitarian Committee

  2. Tabernacle of New Dispensation

  3. Indian Reform Association

Keshab Chandra Sen is associated with the establishment of which of the above?

(a) 1 and 3 only

(b) 2 and 3 only

(c) 3 only

(d) 1, 2 and 3

Ans: b

Sub-Theme: Social Reform Movement/Social Reformist

Keshab Chandra Sen (1838-1884):

  • Appointed as Acharya of the Brahmo Samaj in 1858.
  • Spread the ideas of the Samaj outside Bengal (in United Provinces, Bombay, Punjab, etc.).
  • Showed radical views against caste system, favored inter-caste marriage.
  • Expelled from Acharya’s position in
  • Founded Brahmo Samaj of India (1866).
  • Ananda Mohan Bose, Shibchandra Deb, and Umesh Chandra Datta created the Sadharan Brahmo Samaj in opposition to his ideas.
  • Keshab Chandra Sen erected the mandir known as the Tabernacle of New Dispensation on January 24, 1868, in honour of the Magha Festival..
  • On October 29, 1870, the Indian Reform Association was founded with Keshab Chandra Sen as President. It comprised many people who did not belong to the Brahmo Samaj and represented the secular wing of the organisation.
  • The objective was to put into effect some of the concepts Sen was exposed to while in Britain.

 

2015

 

Question 1

Who of the following was/were economic critic/ critics of colonialism in India?

  1. Dadabhai Naoroji

  2. Subramania Iyer

  3. C. Dutt

Select the correct answer using the codes given below.

(a) 1 only

(b) 1 and 2 only

(c) 2 and 3 only

(d) 1, 2 and 3

Ans: d

Sub-Theme: Economic Exploitation of the British/Economic critics of colonialism

Dadbhai Naoroji, Subramania Iyer and R.C. Dutt are considered as the economic critics of colonialism in India.

Nationalists Critique Of Colonial Economy:

  • The early nationalist of India analysed the colonial character of British rule in India in great detail. The economic critique produced by them was one of the remarkable achievements of the moderate nationalists.
  • Dadabhai Naoroji (Grand Old man of India): “Poverty and UnBritish Rule in India” – Propounded Drain of Wealth Uncovered the economic exploitation of India by the British.
  • Romesh Chandra Dutta: “The Economic History of India” – Half of the annual GDP of India outflowed every year.
  • V. Joshi: Pointed out that the development in India was like a subsidy to the British Industries.
  • Other Critiques: Justice Mahadev Govind Ranade, G. Subramaniam Iyer, G.K. Gokhale etc. examined every facet of the economy and subjected the entire range of economic issues and colonial economic policies to minute scrutiny.
NOTE: This question can be made correct by reading Modern NCERT. It’s always recommended reading between the lines about every personality.

 

Question 2

The Government of India Act of 1919 clearly defined:

(a) The separation of power between the judiciary and the legislature

(b) The jurisdiction of the central and provincial governments

(c) The powers of the Secretary of State for India and the Viceroy

(d) None of the above

Ans: b

Sub-Theme: Administrative Reforms post 1857 The Montagu-Chelmsford Reforms which became the Government of India Act in 1919, defined the jurisdiction of the central and provincial governments.

  • The GoI Act 1919 introduced a diarchy (rule of two individuals/parties) for the executive at the level of the provincial
  • The diarchy was implemented in eight provinces: Assam, Bengal, Bihar and Orissa, Central Provinces, United Provinces, Bombay, Madras and Punjab.
  • The Dyarchy system increased the authority of the provincial governments.
  • In the province, the governor was to serve as the chief executive.

Introduction to Diarchy:

  • The GoI Act 1919 introduced a diarchy (rule of two individuals/parties) for the executive at the level of the provincial government.
  • The diarchy was implemented in eight provinces: Assam, Bengal, Bihar and Orissa, Central Provinces, United Provinces, Bombay, Madras and Punjab.
  • The Dyarchy system increased the authority of the provincial governments.
  • In the province, the governor was to serve as the chief executive.

Division of Subjects:

  • Division of subjects into two lists: ‘reserved’ and ‘transferred’.
  • The reserved list: included subjects such as law and order, finance, land revenue, irrigation and the subjects were to be administered by the governor through his executive council of bureaucrats. All significant subjects were maintained in the Provincial Executive’s reserved subjects.
  • The transferred subjects: were to be administered by ministers nominated from among the elected members of the legislative council and it included subjects such as education, health, local government, industry, agriculture, excise, etc.

 

Question 3

Which one of the following movements has contributed to a split in the Indian National Congress resulting in the emergence of ‘moderates’ and ‘extremists’?

(a) Swadeshi Movement

(b) Quit India Movement

(c) Non-Cooperation Movement

(d) Civil Disobedience Movement

Ans: a

Sub-Theme: Swadeshi Movement

Swadeshi and Boycott Movements:

  • It was launched due to the partition of Later, the movement spread to other regions of the nation:
    • Poona and Bombay: Bal Gangadhar Tilak
    • Punjab: Lala Lajpat Rai, Ajit Singh
    • Delhi: Syed Haider Raza
    • Madras: Chidambaram Pillai
  • The Swadeshi Movement contributed to a split in the Indian National Congress in 1907, resulting in the emergence of ‘moderates’ and ‘extremists’.
  • Congress Session 1907, Surat, presided by Rash Behari Ghosh was known as Surat split.
  • Extremists wanted either G. Tilak or Lajpat Rai as President.
  • Moderates proposed Rashbehari Ghosh and wanted the session to be in Surat in order to exclude Tilak from the presidency.
  • Since a leader from the host province could not be session president, they also sought to drop resolutions on swadeshi, boycott and national education.

 

Question 4

With reference to Rowlatt Satyagraha, which of the following statements is/are correct ?

  1. The Rowlatt Act was based on the recommendations of the ‘Sedition Committee’.

  2. In Rowlatt Satyagraha, Gandhiji tried to utilise the Home Rule League.

  3. Demonstrations against the arrival of the Simon Commission coincided with Rowlatt Satyagraha.

Select the correct answer using the codes given below.

(a) 1 only

(b) 1 and 2 only

(c) 2 and 3 only

(d) 1, 2 and 3

Ans: b

Sub-Theme: Phase of Mahatma Gandhi

Statement 1 is correct: Rowlatt Act was passed on the recommendations of the Sedition Committee chaired by Sir Sidney Rowlatt.

Statement 2 is correct: In organising his satyagraha, Gandhiji tried to utilise three types of political networks: The Home Rule Leagues; Certain Pan-Islamist groups and Satyagraha Sabha.

Statement 3 is incorrect: The two time periods are not the same. It has a gap of 9 years, the incident of Rowlatt Satyagraha took place in 1919 and the Demonstrations against the arrival of Simon Commission took place in 1928, thus the two events cannot coincide.

 

NOTE: Question on Rowlatt act came in 2012 as well. Moreover, the Rowlatt Satyagraha took place in 1919 and the Simon Commission in 1928. So the possibility of coincidence of Simon Commission with Rowlatt Satyagraha stands null. Point to note here is that the relative idea of timeline and corresponding events is very important. This helps us to solve or eliminate statements.

 

Question 5

With reference to Congress Socialist Party, consider the following statements:

  1. It advocated the boycott of British goods and evasion of taxes.

  2. It wanted to establish the dictatorship of the proletariat.

  3. It advocated separate electorate for minorities and oppressed classes.

Which of the statements given above is/are correct?

(a) 1 and 2 only

(b) 3 only

(c) 1, 2 and 3

(d) None

Ans: d

Sub-Theme: Congress Socialist Party

Statement 1 is incorrect: On May 17, 1934, at Patna, with Acharya Narendra Dev as president and Jayaprakash Narayan (JP) as general secretary, the Congress Socialist Party (CSP) was founded within the Congress. The CSP advocated decentralised socialism in which co-operatives, trade unions, independent farmers, and local authorities would hold a substantial share of the economic power. It did not advocate.

Statement 2 is incorrect: Their focus was to attain independence and believed in socialism through nationalism. The CSP promoted a decentralised socialist system in which local authorities, cooperatives, trade unions, and independent farmers would control a sizable portion of the economic power. Hence, it did not aim to establish the dictatorship of the proletariat.

Statement 3 is incorrect: As secularists, they hoped to transcend communal divisions through class solidarity. Many people, like Narendra Deva and Basawon Singh (Sinha), favoured a democratic socialism that was separate from both reformist social democracy and Marxism.

 

Question 6

Who of the following organised a march on the Tanjore coast to break the Salt Law in April 1930?

(a) O. Chidambaram Pillai

(b) Rajagopalachari

(c) Kamaraj

(d) Annie Besant

Ans: b

Sub-Theme: Phases of Mahatma Gandhi/ SatyagrahaMovements/Salt Satyagraha

Civil Disobedience Movement – The Salt Satyagraha and other upsurges:

Dandi March (March 12- April 6, 1930):

  • The Dandi march was led by Gandhi and began at Sabarmati Ashram and ended close to the Dandi village’s coastal location; it was also known as Salt Satyagraha.
  • He reached Dandi on April 6, 1930, and broke the salt law by collecting salt from the beach.

Satyagraha at Different Places:

  • Tamil Nadu: C Rajagopalachari organised a march from Tiruchirapalli to Vedaranniyam on Tanjore coast.
  • Malabar: K. Kelappan known for Vailkom Satyagraha organised salt marches.
  • Orissa: Gopal  Bandhu  Chaudhuri organised in Balasore, Cuttack etc.
  • Bihar: Non chowkidari tax was
  • Peshawar: Badshah khan/khan Abdul Gaffar Khan(Frontier Gandhi) who organised Khudaikhidmatgars (red shirts) organised a powerful march.
  • Dharasana: Sarojini Naidu led the
  • Nagaland: Rani Gaidinliu at the age of 13 years raised the banner against the British

 

 

Question 7

With reference to the Cabinet Mission, which of the following statements is/are correct?

  1. It recommended a federal government.

  2. It enlarged the powers of the Indian courts.

  3. It provided for more Indians in the ICS.

Select the correct answer using the code given below.

(a) 1 only

(b) 2 and 3

(c) 1 and 3

(d) None of these

Ans: a

Sub-Theme: Cabinet Mission

Statement 1 is correct: On 22nd January 1946, the decision to send Cabinet Mission was taken and on 19th February 1946, the British PM C.R Attlee Government announced in the House of Lords about the mission and the plan to quit India. A high-powered mission of three British Cabinet members- Lord Pethick-Lawrence(the Secretary of State for India), Sir Stafford Cripps (President of the Board of Trade) and A. V. Alexander(the First Lord of the Admiralty) reached Delhi on 24th March 1946. All subjects other than Union subjects and residuary power would vest in the provinces of British India. Thus, the Cabinet Mission plan proposed a weak Centre with Provincial autonomy, essentially proposing a federal structure of government.

Statements 2 and 3 are incorrect: The plan did not include the enlargement of the powers of the Indian Court nor provided for more Indians in the ICS.

Objective of the Cabinet Mission:

  • To resolve political deadlock between the Indian National Congress and the All-India Muslim League.
  • The Congress party wanted to obtain a strong central government with more powers as compared to state governments.
  • All India Muslim League under Jinnah wanted to keep India united but with political safeguards provided to Muslims such as ‘guarantee’ of ‘parity’ in the legislatures.
  • On 16 May 1946, this plan was announced and preceded by the Shimla Conference of
  • Recommendation:
    • The unity of India had to be retained
    • It proposed a very loose union of all the Indian territories under a centre that would control merely defence, the Foreign Affairs and the Communication. The Union would have the powers necessary to raise the finances to manage these subjects.
    • All subjects other than Union subjects and residuary power would vest in the provinces of British India. Thus, the Cabinet Mission plan proposed a weak Centre with Provincial autonomy, essentially proposing a federal structure of government.
    • The Princely Legislatures would then elect a Constituent Assembly or a Constitution making body with each province being allotted a specified number of seats proportionate to its population.
    • The proposed Constituent Assembly was to consist of 292 members from British India and 93 from Indian States.
    • The Mission proposed an immediate formation of Interim Government at the centre, enjoying the support of major political parties and with the Indians holding all the portfolios.

 

Question 8

Consider the following statements:

  1. The first woman President of the Indian National Congress was Sarojini Naidu.

  2. The first Muslim President of the Indian National Congress was Badruddin Tyabji.

Which of the statements given above is/are correct?

(a) 1 only

(b) 2 only

(c) Both 1 and 2

(d) Neither 1 nor 2

Ans: b        

Sub-Theme: Important Personalities/ Important sessions of INC

Statement 1 is incorrect: The first woman President of the Indian National Congress was Annie Besant in the 1917 Calcutta Session. Sarojini Naidu became the first Indian woman President of INC in 1925 Kanpur Session.

Statement 2 is correct: Badruddin Tyabji became the first Muslim President of the Indian National Congress in the 1887 Madras Session.

 

2014

 

Question 1

What was/were the object/objects of Queen Victoria’s Proclamation (1858)?

  1. To disclaim any intention to annex the Indian States

  2. To place the Indian administration under the British Crown.

  3. To regulate East India Company’s trade with India

Select the correct answer using the code given

(a) 1 and 2 only

(b) 2 only

(c) 1 and 3 only

(d) 1, 2 and 3

Ans: a

Sub-Theme: Administrative Reforms post 1857/ Queen’s Proclamation

Queen Victoria’s Proclamation (1858):

  • The proclamation was read out by Lord Canning at a durbar held on November 1, 1858 at Allahabad. It put an end to the rule of the East India Company and the government of India came directly under the Crown.
  • According to the proclamation Lord Canning became the first viceroy and governor general of India.
  • The proclamation put an end to the era of further expansion of the British Empire in India.

Government 0f India Act (1858)

  • It was known as the “Act of Good Government of India”.
  • According to its provisions, Her Majesty’s government would now be in charge of India and would do so in her name.
  • It abolished the East India Company and transferred powers to the Crown.

Statement 1 is correct: The proclamation was read out by Lord Canning at a durbar held on November 1, 1858 at Allahabad. It put an end to the rule of the East India Company and the government of India came directly under the Crown. According to the proclamation Lord Canning became the first viceroy and governor general of India. The proclamation put an end to the era of further expansion of the British Empire in India.

Statement 2 is correct: Queen Victoria’s proclamation of 1858 aimed at ending the rule of the company and transferring all the powers to the British crown after the 1857 revolt. It also promised non- interference in the religious affairs of the people, grant of equal protection of law and respect for ancient rights and customs of the people.

Statement 3 is incorrect: Queen Victoria’s Proclamation (1858) is aimed at improving the administrative machinery of supervision and control over the Indian government.

 


 

Question 2

The Partition of Bengal made by Lord Curzon in 1905 lasted until:

(a) The First World War when Indian troops were needed by the British and the partition was ended.

(b) King George V abrogated Curzon’s Act as the Royal Durbar in Delhi in 1911

(c) Gandhiji launched his Civil Disobedience movement.

(d) The Partition of India in 1947 when East Bengal became East Pakistan.

Ans: b

Sub-Theme: Partition of Bengal

Partition of Bengal (1905):

  • The decision to divide Bengal was made public by the British Government in 1903 and partition came into force in October 1905.
  • Reasons given by British- Bengal are too big to be administered and this division of Bengal into eastern and western Bengal would develop Assam.
  • Moderates started anti partition campaigns:
  • Leaders: Surendranath Banerjee, K Mitra and Prithwishchandra Ray
  • Newspaper: Hitabadi, Sanjibani,
  • The Partition of Bengal was annulled during the visit of the Royal Majesties, George V and Queen Mary to India. Partition of Bengal was annulled mainly to curb the rising revolutionary extremism/terrorism. This action disappointed the Muslim political King George V announced the abrogation in the Delhi Durbar on 11th December, 1911. King George V abrogated Curzon’s Act as the Royal Durbar in Delhi in 1911.


 

Question 3

The Ghadr (Ghadar) was a:

(a) revolutionary association of Indians with headquarters at San Francisco.

(b) nationalist organisation operating from Singpore.

(c) militant organisation with headquarters at Berlin

(d) Communist movement for India’s freedom with head-quarters at Tashkent.

Ans: a

Sub-Theme: Revolutionary Activities Abroad

About the Ghadar:

  • The Ghadar Party was a revolutionary organisation centred on the weekly newspaper “The Ghadar,” which had branches in the Far East and along the US coast in addition to its San Francisco
  • Ramdas Puri, D. Kumar, Taraknath Das, Sohan Singh Bhakna, and Lala Hardayal, who arrived there in 1911, were involved in pre-Ghadar revolutionary activities.
  • The Ghadar programme aimed to organise official assassinations, disseminate anti- imperialist and revolutionary literature, coordinate with Indian troops stationed abroad, get weapons, and spark a simultaneous uprising across all British colonies
  • The Ghadar Party’s inspirational forces were Lala Hardayal, Ramchandra, Bhagwan Singh, Kartar Singh Saraba, Barkatullah, and Bhai Parmanand.
  • The Ghadarites intended to bring about a revolt in Their plans were encouraged by two events in 1914, the Komagata Maru incident and the outbreak of the First World War.


 

Question 4

The 1929 Session of Indian National Congress is on significance in the history of the Freedom Movement because the:

(a) attainment of Self-Government was declared as the objective of the Congress.

(b) Attainment of Poorna Swaraj was adopted as the goal of the Congress.

(c) Non-Cooperation Movement was launched.

(d) Decision to participate in the Round Table Conference in London was taken.

Ans: b

Sub-Theme: Important Sessions of INC

Option (a) is incorrect: Attainment of Self- Government was declared as the objective of the Congress in the Calcutta Session, 1906.

Option (b) is correct: Attainment of Poorna Swaraj was adopted as the goal of the Congress in the Lahore Session, 1929 under the presidentship of Jawaharlal Nehru. Along with this, the Congress made the decision to abstain from the First Round Table Conference because the British Government declined to call a Constituent Assembly to draft a constitution for India. Launch of a civil disobedience movement for complete independence and 26 January to be observed as ‘Independence Day’/Swarajya Day were also decided in the Lahore Session.

Option (c) is incorrect: Mahatma Gandhi moved the Non-cooperation resolution in the Special Calcutta Session of Congress in 1920.

Option (d) is incorrect: Decision to participate in the Round Table Conference in London was taken in the Karachi Session of Congress in 1931.

NOTE: Question on the Lahore Session of the Indian National Congress (1929) earlier came in 2012. This particular theme is a high-yielding theme and if the aspirants cover this theme/sub-theme diligently then questions from this section may appear as low-hanging fruits which an aspirant cannot miss.


 

Question 5

The Radcliffe Committee was appointed to:

(a) Solve the problem of minorities in India

(b) Given effect to the Independence Bill

(c) Delimit the boundaries between India and Pakistan

(d) Enquire into the riots in East Bengal

Ans: c

Sub-Theme: Last phase of Indian Freedom Struggle/Boundary Commission/Mountbatten Plan/Transfer of Power

  • In the event of Partition, a boundary commission led by Sir Cyril Radcliffe was established which was to delimit the boundaries between India and Pakistan. Boundary Commission, consultative committee created in July 1947 to recommend how the Punjab and Bengal regions of the Indian subcontinent were to be divided between India and Pakistan shortly before each was to become independent from Britain.

 

2013

 

Question 1

The Ilbert Bill controversy was related to the:

(a) Imposition of certain restrictions to carry arms by the Indians

(b) Imposition of restrictions on newspapers and magazines published in Indian languages

(c) Removal of disqualifications imposed on the Indian magistrates with regard to the trial of the Europeans

(d) Removal of a duty on imported cotton cloth

Ans: c

Sub-Theme: Ilbert Bill controversy

Ilbert Bill Controversy (1883):

  • Two types of law that had been common in India were what Lord Ripon sought to abolish.
  • The legal system stipulated that a European could only be tried by a European Judge or a European Magistrate.
  • The disqualification was unfair, because it was intended to disgrace and discredit the judges who were of Indian descent.
  • A bill to end this prejudice in the judiciary was proposed in 1883 by lawmaker C.P. Ilbert. Europeans were vehemently against this Bill.
  • The Ilbert Bill controversy made educated Indians aware of the need of an all Indian organisation, resulting in the formation of Indian National Congress in 1885 in Bombay with early leadership of Dadabhai Naroji, Pherozeshah Mehta, Badruddin Tyabji, WC Banerjee etc.
  • The Ilbert Bill Controversy is a high watermark in the history of the Indian National Movement. The immediate result of this awakening of India was the birth of the Indian National Congress in 1885, the very next year of Ripon’s departure.

Option (a) is incorrect: The Indian Arms Act (1878) imposed restrictions and it compelled the Indians to have license to keep, sell or purchase arms.

Option (b) is incorrect: The Vernacular Press Act (1878) posed restrictions on newspapers and magazines published in Indian languages.

Option (c) is  correct:  The  Ilbert  Bill Controversy 1883 was related to the removal of disqualifications imposed on the Indian magistrates with regard to the trial of the Europeans.

Option (d) is incorrect: On March 13th, 1879, Lord Lytton abolished the import duty on cotton goods except on the finest qualities and it has no relation with the Ilbert Bill controversy.

 

Question 2

The demand for the Tebhaga Peasant Movement in Bengal was for:

(a) the reduction of the share of the landlords from one-half of the crop to one third.

(b) the grant of ownership of land to peasants as they were the actual cultivators of the land.

(c) the uprooting of Zamindari system and the end of serfdom.

(d) writing off all peasant debts.

Ans: a

Sub-Theme: Peasant Movement

Tebhaga Peasant Movement (1946) in Bengal demanded the share of the landlords to be reduced from one-half of the produce to one third of the produce.

Tebhaga Movement (1946):

  • In September 1946, the Bengal Provincial Kisan Sabha gave a call to implement, through mass struggle, the Floud Commission recommendations of Tebhaga which means two-thirds’ share to the bargardars, the share-croppers also known as ‘bhagchasi’ or ‘adhyar’, instead of the one-half share.
  • The jotedars rented the land that the bargardars worked on.
  • The main slogan was “nij khamare dhan tolo,” which meant that sharecroppers should thresh their own paddy and not to the jotedar’s house, as before, so as to enforce tebhaga.
  • North Bengal was the epicentre of the movement, particularly among the Rajbanshis, a low caste with tribal origins. Muslims took part in great numbers as well.
  • The movement dissipated soon, as the League ministry’s sop of the Bargardari Bill, the popularisation of the Hindu Mahasabha’s campaign for a separate Bengal, and new riots in Calcutta, which eliminated the possibility of sympathetic support from the urban sections.
NOTE: If we read or Modern NCERT carefully, Tebhaga literally means ‘three shares’ of harvests. Option 1 talks about the “one-third” part. With just knowing the meaning of “Tebhaga’’, we can solve the given question.

 

Question 3

The people of India agitated against the arrival of the Simon Commission because:

(a) Indians never wanted the review of the working of the Act of 1919

(b) Simon Commission recommended the abolition of Dyarchy (Diarchy) in the Provinces

(c) There was no Indian member in the Simon Commission

(d) The Simon Commission suggested the partition of the country

Ans: c

Sub -Theme: Administrative Refor m/ Constitutional Reforms

Simon Commission (1927):

  • The Indian Statutory Commission, commonly referred to as the Simon Commission, was a group of seven British Members of Parliament under the chairmanship of Sir John Simon who arrived in British India in
  • Objective: The Simon Commission was appointed by the British government to assess India’s constitutional development and recommend constitutional reforms.
  • Many Indians strongly opposed the Commission for a variety of reasons. The main reason behind the agitation against Simon Commission was that there were no Indian members of the panel and only seven British members of the British Parliament. This was viewed as racist and colonist.

 

Question 4

Quit India Movement was launched in response to:

(a) Cabinet Mission Plan

(b) Cripps Proposals

(c) Simon Commission Report

(d) Wavell Plan

Ans: b

Sub-Theme: Quit India Movement

The failure of the Cripps Mission to resolve the constitutional deadlock made it clear to the nationalists that any silence would amount to accepting the right of the British to decide the fate of Indians without consulting them.

  • Quit India Movement was launched in response to Mahatma Gandhi’s national call for satyagraha. The All-India Congress Committee proclaimed a mass protest demanding what Gandhiji called “an orderly British withdrawal” from India.
  • The Quit India resolution was ratified at the Congress meeting at Gowalia Tank, Bombay on August 8, 1942.

 

Question 5

Annie Besant was:

  1. Responsible for starting the Home Rule Movement

  2. The founder of the Theosophical Society

  3. Once the President of the Indian National Congress

Select the correct statement/statements using the codes given below.

(a) 1 only

(b) 2 and 3 only

(c) 1 and 3 only

(d) 1, 2 and 3

Ans: c

Sub-Theme: Important Personalities

Annie Besant 1847-1933:

  • She fought for the freedom of thought, secularism, women’s rights, birth control, workers’ rights and Fabian socialism.
  • Following her encounter with Helena Blavatsky in 1889, Besant became a Theosophist.
  • In 1893, she made her first trip to India with the Theosophical Society.
  • She and Henry Steel Olcott co-founded the Theosophical Society – Adyar.
  • She was the society’s president from 1907 to 1933.
  • In Benares, Besant founded the Central Hindu College (CHC).
  • She joined the Indian National Congress (INC) and demanded government action towards self-rule. She became the President of the Calcutta Session of INC, AD 1917.
  • In 1916, Besant and Bal Gangadhar Tilak founded the All-India Home Rule League.
  • She did not attend the AD 1920 Session at Nagpur due to growing differences with Gandhiji as she felt that the Government of India Act, 1919 was a means to free India.
  • Newspapers: New India and Commonweal.
  • She wrote the ‘Lotus Song’, an English version of the Gita.

Statement 1 is correct: Annie Besant fought for  the  freedom  of  thought,  secularism, women’s rights, birth control, workers’ rights and Fabian socialism. In 1916, Annie Besant and Tilak started the Home Rule Leagues at Madras and Belgaum respectively.

Statement 2 is incorrect: The Theosophical Society  was  founded  by  Madame  H.  P. Blavatsky and Colonel Olcott in New York in 1875. In 1882, the headquarters of the Society were established in Adyar, near Madras (now Chennai) in India.

Statement 3 is correct: Annie Besant became the President of the INC in Calcutta Session,1917.

 

Question 6

With reference to Indian History, the Members of the Constituent Assembly from the Provinces were:

(a) directly elected by the people of those Provinces

(b) nominated by the Indian National Congress and the Muslim League

(c) elected by the Provincial Legislative Assemblies

(d) selected by the Government for their expertise in constitutional matters

Ans: c          

Sub-Theme: Constitutional Development

The idea of a constituent assembly was put forward for the first time by MN Roy. The Indian National Congress (INC) first formally requested a constituent assembly to draft an Indian constitution in 1935. Representatives of each community were to be elected by members of that community in the provincial legislative assembly and voting was to be by the method of proportional representation by means of single transferable vote. Composition of the Constituent Assembly:

  • C.A. constituted in Nov 1946 under CMP (Cabinet Mission Plan).
  • Seats allotted as per population proportion.
  • Voting using a single transferable vote and the proportional representation system when a province is represented.
  • Mahatma Gandhi was not part of C.A.
  • Strength of the assembly:
    • Total strength of the assembly: 389
    • 296 seats for British India and 93 seats to princely states
    • 292 seats allocated for British India were to be from eleven governor’s provinces and four from Chief commissioner’s provinces.
    • Seats were allocated based on proportion to their respective population.
    • Muslims, Sikhs, and general voters were to determine which of the three major populations in each British province would hold the seats up for election in the provincial legislative assembly.
    • The heads of these princely states were supposed to nominate the representatives.

 

2012

 

Question 1

With reference to Ryotwari Settlement, consider the following statements:

  1. The rent was paid directly by the peasants to the government.

  2. The government gave Pattas to the Rytos.

3. The lands were surveyed and assessed before being taxed.

Which of the statements given above is/are correct?

(a) 1 only

(b) 1 and 2 only

(c) 1, 2 and 3

(d) None

Ans: c

Sub-Theme: Land Revenue Settlement

  • Statements 1 and 2 are correct: The Ryotwari System was started by Thomas Munro and Alexander Reed. It was started in the Baramahal district of Tamil Nadu and later spread to Madras, parts of Bombay, East Bengal, Assam and Kurg (Karnataka). It covered 51% of British India. Ryots (farmers) were given the ownership and other rights (Pattas) over the land and were required to pay the revenue directly to the government.
  • Statement 3 is correct: The Ryotwari System involved high taxes (50% in dryland and 60% in wetland) and strict collection.
  • Mahalwari System.1 2
NOTE: Question on land settlement system came in 2011 – on land settlement system of Lord Cornwallis.

 

Question 2

Consider the following statements:

The most effective contribution made by Dadabhai Naoroji to the cause of Indian National Movement was that he-

1. Exposed the economic exploitation of India by the British.

2. Interpreted the ancient Indian texts and restored the self-confidence of Indians.

3. Stressed the need for eradication of all the social evils before anything else.

Which of the statements given above is/are correct?

(a) 1 only

(b) 2 and 3 only

(c) 1 and 3 only

(d) 1, 2 and 3

Ans: (a) Sub-Theme: Economic Exploitation of the British/Economic critics of colonialism

Statement 1 is correct: Dadabhai Naoroji (Grand Old man of India) wrote the “Poverty and UnBritish Rule in India” a Propounded Drain of Wealth Theory and exposed the economic exploitation of India by the British.

Statement 2 is incorrect: Swami Dayanand Saraswati, Swami Vivekananda, etc. interpreted the ancient texts and restored the self confidence of Indians, not Dadabhai Naoroji. Swami Dayananda Saraswati even gave the call to “go back to vedas”.

Statement 3 is incorrect: Raja Rammohan Roy, Ishwar Chandra Vidyasagar, Keshav Chandra Sen, Atmaram Pandurang, etc. emphasised the need for eradication of all social evils before anything else. Dadabhai Naoroji wasn’t a social reformer.

NOTE: If we carefully look at statement 3, it talks about the “eradication of all the social evils”. This sounds absolute and exclusive because of the extreme word “all”. Eliminating all social evils was a mammoth and uphill task. So there is less possibility of correctness of this statement. If we take calculable risk of eliminating statement 3, we can get an answer. Moreover, Dadabhai Naoroji propounded “Drain of wealth theory”. It was about exposing the economic exploitation of India by the British. This makes statement 1 correct. Economic Exploitation of the British/ Economic critics of colonialism theme is significant with respect to Colonial history in India. In the past, many questions came from this particular topic to check the analytical ability of aspirants.


 

Question 3

Consider the following:

  1. Assessment of land revenue on the basis of nature of the soil and the quality of crops.

  2. Use of mobile cannons in warfare.

  3. Cultivation of tobacco and red chillies.

Which of the above was/were introduced into India by the English?

(a) 1 only

(b) 1 and 2

(c) 2 and 3

(d) None

Ans: d

Sub-Theme: Colonial Trade + Land Revenue Settlement

Statement 1 is incorrect: After getting the Diwani rights of Bengal, Bihar, and Orissa in 1765 the major aim of the British East India Company was to increase their land revenue collection. So its policies were aimed at getting maximum income from land without caring of its consequences on cultivators and peasants. Three major systems of land revenue collection existed in India – Zamindari, Ryotwari and Mahalwari.

Statement 2 is incorrect: The first recorded use of Artillery in India is at the Battle of Adoni in 1368. In the Deccans the Bahamani kings led by Mohammand Shah Bahamani used a train of Artillery against the Raja of Vijaya-nagar.

Statement 3 is incorrect: Tobacco and Red Chillies were introduced by Portuguese not British.

NOTE: In general, assessment of land revenue on the basis of nature of the soil and the quality of crops itself cumbersome and administratively mammoth task. It also needs a high level of record-keeping practice on part of administration. So this statement sounds. Now, if we carefully go through Spectrum or Modern NCERT it is clear that Portuguese had introduced Tobacco in India.

 

Question 4

Which of the following is/are the principal feature(s) of the Government of India Act, 1919?

  1. Introduction of diarchy in the executive government of the provinces.

  2. Introduction of separate communal electorates for Muslims.

  3. Devolution of legislative authority by the centre to the provinces.

Select the correct answer using the codes given below:

(a) 1 only

(b) 2 and 3 only

(c) 1 and 3 only

(d) 1, 2 and 3

Ans: c

Sub-Theme: Administrative Reform post-1857 Statements 1 and 3 are correct: GoI Act 1919 introduced the dual scheme of governance known as  ‘dyarchy’  in  the  executive government of the provinces. By demarcating and separating the central and provincial subjects, it relaxed the central control over the provinces. It introduced bicameralism and direct elections in the country. The devolution of legislative authority by the centre to the provinces was done in the Government of India Act, 1919.

Statement 2 is incorrect: The Indian Council Act, 1909 or the Morley-Minto Reforms introduced separate communal electorates for Muslims. It was put into place to appease the moderates (in Congress) and introduces separate electorates based on religion. Therefore, Lord Minto came to be known as “Father of the Communal Electorate in India”.

 

Question 5

The distribution of powers between the Centre and the States in the Indian Constitution is based on the scheme provided in the:

(a) Morley-Minto Reforms, 1909

(b) Montagu-Chelmsford Act, 1919

(c) Government of India Act, 1935

(d) Indian Independence Act, 1947

Ans: c

Sub-Theme: Administrative Reform post 1857

Government of India Act, 1935:

  • Divided the powers between the Centre and provinces in terms of three lists– Federal List (for Centre, with 59 items), Provincial List (for provinces, with 54 items) and the Concurrent List (for both, with 36 items).
  • Abolished dyarchy in the provinces and introduced ‘provincial autonomy’ in its place, while adopted the dyarchy at the Centre
  • Introduced bicameralism in six out of eleven provinces
  • Further extended the principle of communal representation to the depressed classes (Scheduled Castes), women and labour (workers).
  • Abolished the Council of India, established by the Government of India Act of 1858.

NOTE: In every alternate year, in one way or another there is a question on acts and policies of colonial times. It is advisable to go through each and every act/regulation/ law in detail and provision by provision.

 

Question 6

During Indian freedom struggle, the National Social Conference was formed. What was the reason for its formation?

(a) Different social reform groups or organizations of the Bengal region united to form a single body to discuss the issues of larger interest and to prepare appropriate petitions/representations to the government.

(b) Indian National Congress did not want to include social reforms in its deliberations and decided to form a separate body for such a purpose.

(c) Behramji Malabari and M.G. Ranade decided to bring together all the social reform groups of the country under one organization.

(d) None of the statements (a), (b) and (c) given above is correct in this context.

Ans: b

Sub-Theme: Social Reform Organizations

Indian Social Conference (1887):

  • It was formed by M. G. Ranade and Raghunath Rao in Madras.
  • It is a separate social reform organ of the Indian National Congress as they did not want to include social reforms in their
  • Advocated inter-caste marriage, opposed polygamy and kulinism.
  • Launched “Pledge Movement to pledge against Child Marriage”

 

Question 7

Which of the following statements is/are correct regarding Brahmo Samaj?

  1. It opposed idolatry.

  2. It denied the need for a priestly class for interpreting the religious texts.

  3. It popularized the doctrine that the Vedas are infallible.

Select the correct answer using the codes given below:

(a) 1 only

(b) 1 and 2 only

(c) 3 only

(d) 1, 2 and 3

Ans: b

Sub-Theme: Social Reform Organisations/ Social Reformist

Brahmo Samaj did oppose idolatry and denied the need for a priestly class for interpreting the religious texts.

BRAHMO SAMAJ (1828)

  • Raja Ram Mohan Roy founded the Brahmo Samaj (1828) [formerly called Brahmo]
  • Later Debendranath Tagore joined the Brahmo Samaj in 1842.
  • Keshab Chandra Sen was appointed as Acharya of the Brahmo Samaj in 1858.
  • Favoured: Human reason and conscience; Political upliftment of the masses, widow
  • Against: Polytheism; Incarnations; Scriptural authority; Idol worship; Caste system; Untouchability; Sati; Child marriage; Purdah system.
  • No definite view: on Karma and the transmigration of the soul.
  • Punjab: Dyal Singh College at Lahore (1910) started by Dayal Singh Trust, popularised the ideas of the Samaj.

Statement 1 is correct: Raja Ram Mohan Roy founded the Brahmo Samaj (1828) [formerly called Brahmo Sabha.] Later Debendranath Tagore joined the Brahmo Samaj in 1842. Brahmo Samaj vehemently opposed idolatry or idol worship.

Statement 2 is correct: The chief aim of Brahmo Samaj was the worship of the eternal God. It was against priesthood, rituals and sacrifices.

Statement 3 is incorrect: Brahmo Samaj spread the religious treasure and teachings of Vedas but never claimed that Vedas are infallible.

 

Question 8

The Rowlatt Act aimed at:

(a) Compulsory economic support to war efforts

(b) Imprisonment without trial and summary procedures for trial

(c) Suppression of the Khilafat movement.

(d) Imposition of restrictions on freedom of the press.

Ans: b

Sub-Theme: Phase of Mahatma Gandhi/Acts and Regulation under the Crown Rule

Option (a) is incorrect: The drain on the Indian economy in the form of cash, kind and loans to the British government came to about 367 million pounds during the WW-I. Rowlatt Act has no relation with draining Indian money or economic support to war efforts.

Option (b) is correct: Anarchical and Revolutionary Crimes Act/ Rowlatt act gives the notorious power to imprison activists without trial for two years, even possession of seditious newspapers is adequate evidence of guilt.

Option (c) is incorrect: The Khilafat Movement (1919-20) was a movement to express Muslim support for the Caliph of Turkey, to protect the institution of the Khalifa in Turkey, against the allied powers, particularly Britain. The Rowlatt Act was not aimed at suppressing the Khilafat Movement.

Option (d) is incorrect: Vernacular Press Act, 1878, The Newspaper Act, 1908 and The Indian Press Act, 1931 were passed in order to impose restrictions on freedom of press. It has no relation with the Rowlatt Act.


 

Question 9

The Lahore Session of the Indian National Congress (1929) is very important in history, because:

  1. The Congress passed a resolution demanding complete independence.

  2. The rift between the extremists and moderates was resolved in that Session.

  3. A resolution was passed rejecting the two-nation theory in that Session

Which of the statements given above is/are correct?

(a) 1 only

(b) 2 and 3

(c) 1 and 3

(d) None of the above

Ans: a

Sub-Theme: Important Sessions of INC

Statement 1 is correct: In the Lahore Session of INC(1929), the resolution on ‘Poorna Swaraj’  or  complete  independence  was passed.

Statement 2 is incorrect: The rift between the extremists and moderates took place in the Surat Session(1907) and the split was resolved in the Lucknow Session 1916.

Statement 3 is incorrect: The Cabinet Mission recommended an undivided India and rejected the demand of the Muslim League for a separate Pakistan. The Cabinet Mission felt that a separate state would contain a larger proportion of the Non-Muslim population and a sizable population of Muslims will be left in India, if two states are declared.

 

Question 10

The Congress ministries resigned in the seven provinces in 1939, because:

(a) The Congress could not form ministries in the other four provinces.

(b) Emergence of a ‘left wing’ in the Congress made the working of the ministries impossible.

(c) There were widespread communal disturbances in their provinces.

(d) None of the statements (a), (b) and (c) given above is correct.

Ans: d

Sub-Theme: Resignation of Congress Ministries

The Congress Ministries: In the early 1937, elections to  provincial  assemblies  were held and Congress Ministries were formed in Bombay, Madras, Central Provinces, United Provinces, Bihar, Orissa, Assam, and NWFP. By 1939, there were internal strifes, opportunism and hunger for power had started surfacing among Congressmen, yet they were able to utilise the council work to their advantage to a great extent and ruled for 28 months.

Resignation of Congress Ministries (1939):

  • The Congress Ministries resigned in protest because the then Viceroy Lord Linlithgow (1936-1944) action of declared India to be belligerent in WW-II without even consulting the Indian people.

 

Question 11

Which of the following parties were established by Dr. B.R. Ambedkar?

  1. The Peasants and Workers Party of India.

  2. All India Scheduled Castes Federation.

  3. The Independent Labour party.

Select the correct answer using the codes given below:

(a) 1 and 2 only

(b) 2 and 3 only

(c) 1 and 3 only

(d) 1, 2 and 3

Ans: b         

Sub-Theme: Important Personalities

Statement 1 is incorrect: The Peasants and Workers Party of India was founded in 1947 The party was founded in Maharashtra by Tulsidas Jadhav, Keshavrao Jedhe and others.

Statement 2 and 3 are correct: Both The Independent Labour Party (1936) and All India Scheduled Castes Federation (1942) were established by Dr. B.R. Ambedkar.

 

Question 12

Mahatma Gandhi undertook fast unto death in 1932, mainly because:

(a) Round Table Conference failed to satisfy Indian political aspirations.

(b) Congress and Muslim League had differences of opinion.

(c) Ramsay Macdonald announced the Communal Award.

(d) None of the statements (a), (b) and (c) given above is correct in this context.

Ans: c     

Sub-Theme: Phases of Mahatma Gandhi

  Communal Award (1932):

  • The Prime Minister of Britain Ramsay MacDonald, announced the Communal Award in August 1932 to pursue the British policy of divide and rule.
  • The Communal Award recognised the depressed classes as minorities and provided them separate electorates.
  • Earlier the British had already given separate electorates to Muslims, Christians and Sikhs.

Poona Pact 1932:

  • B.R. Ambedkar negotiated the Poona Pact with Mahatma Gandhi in late September 1932.
  • It was signed by Madan Mohan Malviya on behalf of Gandhi, and B. R. Ambedkar signed it on behalf of the depressed classes.
  • The background to the Poona Pact was the Communal Award of August 1932, which, among other things, reserved 71 seats in the central legislature for the depressed classes.
  • Gandhi, who opposed the Communal Award and believed it was a British effort to divide Hindus, started a fast unto death in order to have it overturned.
  • He rejected the concept of a special electorate for the depressed classes.
  • However, the number of seats set aside for them in provincial legislatures was increased to 147, and in the Central Legislature, they now make up 18% of all seats.
  • The findings of the Indian Franchise Committee, also known as the Lothian Committee, served as the foundation for the communal award.


 

2011

 

Question 1

The tendency for increased litigation was visible after the introduction of the land settlement system of Lord Cornwallis in 1793. The reason for this is normally traced to which of the following provisions?

(a) Making Zamindar’s position stronger vis-à-vis the ryot

(b) Making East India Company an overlord of

(c) Making judicial system more efficient

(d) None of the (a), (b) and (c)

Ans: d

Sub-Theme: Land Revenue Settlement

Reforms of Cornwallis:

  • The Permanent Land Revenue Settlement of Bengal, which was extended to the provinces of Bihar and Orissa, was Lord Cornwallis’ most notable administrative
  • The reforms had increased the litigation after the introduction of the land settlement system and the main reason for this was the removal of Court Fee and now everybody could drag anybody to the courts.
  • The extension to the right of appeal was also one of the reasons court fees were abolished by Cornwallis and Lawyers were prescribed their fees.
  • Zamindari System:

2 1

  • The Zamindari System was introduced by Lord Cornwallis in 1793 through the Permanent Settlement Act 1793.
  • The land where the zamindars had previously been responsible for collecting revenue was given to them as ownership.
  • The settlement is referred to as the Zamindari Settlement since it was made with the zamindars.
  • Since they were regarded as the soil’s tillers, the ryots were made tenants.
  • The demand for land revenue was predetermined and out of controlled revenue, 10/11th part was given to the
  • Company also added the “Sunset clause” for the zamindars.
NOTE: Many questions came from this particular theme esp. Permanent Settlement, Ryotwari Settlement, Mahalwari Settlement. So understanding differences between them stands crucial.

 

Question 2

With reference to the period of colonial rule in India, “Home Charges” formed an important part of the drain of wealth from India. Which of the following funds constituted “Home Charges”?

  1. Funds used to support the India Office in London.

  2. Funds used to pay salaries and pensions of British personnel engaged in India.

  3. Funds used for waging wars outside India by the British.

Select the correct answer using the codes given below:

(a) 1 only

(b) 1 and 2 only

(c) 2 and 3 only

(d) 1, 2 and 3

Ans: b

Sub-Theme: Economic Exploitation of the British/Economic critics of colonialism

Drain of Wealth and Home Charges:

  • Dadabhai Naoroji (Grand Old man of India) in his “Poverty and UnBritish Rule in India” (1901), propounded the Drain of Wealth Theory.
  • Home Charges played a significant role in the Drain of Wealth from India. It comprised of:
    • Funding for the Indian offices in London.
    • Funds for paying the salaries and pensions of British employees working in India.
    • Funds used for additional capital investments, such as railways, and to pay interest on debts.
  • “Economic history of India” book was written by R. C. Dutta.

Statement 1 is correct: Funds used to supportthe India Office in London. Hence,

Statement 2 is correct: Funds used to pay salaries and pensions of British personnel engaged in India.

Statement 3 is incorrect: No the funds collected as Home Charges were never used for waging wars outside India by the British.

 

Question 3

What was the purpose for which Sir William Wedderburn and W.S. Caine had set up the Indian Parliamentary Committee in 1893?

(a) To agitate for Indian political reforms in the House of Commons

(b) To campaign for the entry of Indians into the Imperial Judiciary

(c) To facilitate a discussion on India’s Independence in the British Parliament

(d) To agitate for the entry of eminent Indians into the British Parliament

Ans: a

Sub-Theme: Administrative Reform post-1857

Together with Dadabhai Naoroji and other supporters of India, Sir William Wedderburn and W.  S.  Caine established the “Indian Parliamentary Committee” in 1893. The Indian Parliamentary Committee’s mission is to lobby the House of Commons for political reforms in India.

About Sir William Wedderburn:

  • President of INC: 1838-1918 (Bombay, 1889, Allahabad, 1910)
  • He came to India in 1860 and registered for work as an Assistant Collector at Dharwad.
  • His interest in these issues led him to contact the Indian National Congress.
  • After retiring, Wedderburn threw himself into He presided over the fourth Congress held in Bombay in 1889.
  • Together with Dadabhai Naoroji and other supporters of India, Sir William Wedderburn and W. S. Caine established the “Indian Parliamentary Committee” in 1893. The Indian Parliamentary Committee’s mission is to lobby the House of Commons for political reforms in India.
  • In 1895, Wedderburn represented India on the Welby Commission (i.e. Royal Commission) on Indian Expenditure.
  • He also began participating in the activities of the Indian Famine Union set up in June 1901, for investigation into famines and proposing preventive measures.
  • He came to India in 1904 to attend the 20th session of the Indian National Congress in Bombay, which was presided over by Sir Henry Cotton.
  • He was again invited in 1910 to preside over the 25th session.
  • He remained the Chairman of the British Committee of the Congress from July 1889 until his death.

NOTE: By a general understanding of modern history, we know congress was established in 1885 and 1893 was a phase of early moderates and demands were mostly concentrated about reforms. Demand for India’s Independence came at a later stage. By this reasoning, we can eliminate option c.

 

Question 4

Which amongst the following provided a common factor for tribal insurrection in India in the 19th century?

(a) Introduction of a new system of land revenue and taxation of tribal products

(b) Influence of foreign religious missionaries in tribal areas

(c) Rise of a large number of money lenders, traders and revenue farmers as middlemen in tribal areas

(d) The complete disruption of the old agrarian order of the tribal communities

Ans: d

Sub-Theme: Tribal Movement

The essence of Tribal life, i.e. shifting cultivation, hunting, fishing and the use of forest produce etc. was completely disrupted during the colonial rule.

Causes of Tribal Revolts in India in the 19th Century:

  • The cornerstone of Tribal life was shifting cultivation, hunting, fishing and the use of forest produce.
  • The practice of settled agriculture was established with the inflow of non-tribals into the traditional tribal lands and as a result of it the tribal population suffered a land loss.
  • There were restrictions imposed on the utilisation of forest products, shifting agriculture, and hunting methods. This led to the loss of livelihood for the tribals.
  • The tribals turned into landless agricultural laboures.
  • Money lenders were introduced by the British into the tribal regions, which greatly increased the exploitation of the native tribes. Under the new economic structure, they were reduced to bonded labour.
  • A joint ownership arrangement for land existed in tribal communities before the idea of private property took hold.
  • Tribal society has historically been egalitarian in contrast to mainstream society, which is characterised by caste and class However, with the arrival of non-tribals or outsiders, tribal people began to be seen as belonging to the lowest strata of society.
  • In order to manage the abundant riches of Indian forests, the government established a Forest Department in 1864. A comprehensive government monopoly on the forested land was created by the Govt Forest Act of 1865 and the Indian Forest Act of 1878.
  • The work of Christian missionaries also led to social unrest in tribal societies, which also aroused their resentment.
  • Major Tribal movements of 19th Century: Kol Rebellion (1832); Santhal Rebellion (1855-56); Munda Rebellion (1899-1900)
NOTE: option (d) sounds more inclusive and wider in coverage in terms of issues faced by tribals. It also includes the meaning and spirit of the above 3 sentences i.e. a, b, and c in one way or other. The options are ambiguous, hence we stuck to the official answer key of UPSC.

 

Question 5

What was the reason for Mahatma Gandhi to organize a satyagraha on behalf of the peasants of Kheda?

  1. The Administration did not suspend the land revenue collection in spite of a drought.

  2. The Administration proposed to introduce Permanent Settlement in Gujarat.

Which of the statements given above is/are correct?

(a) 1 only

(b) 2 only

(c) Both 1 and 2

(d) Neither 1 nor 2

Ans: a

Sub-Theme: Phase of Mahatma Gandhi

Statement 1 is correct: Kheda Satyagraha (1918) was  the first Non-cooperation Movement. Crops failed in Kheda district of Gujarat because of drought. The yield was less than 1/4th of normal produce so farmers were entitled to remission according to revenue code but the Government demanded taxes and ordered seizure of property, if not paid. Mahatma Gandhi asked the farmers not to pay the taxes and organise a satyagraha on behalf of the peasants. Leaders: Sardar Vallabhai Patel, Mohanlal Pandya, Narhari Parekh, etc. Government finally agreed to return all confiscated property, reduce the increase in tax, and suspend the tax for the year.

Statement 2 is incorrect: Permanent Settlement was introduced by Lord Cornwallis in 1893 and it was confined in the areas of Bengal, Bihar, Orissa, Varanasi (United Provinces), Northern Karnataka. It was never introduced in Gujarat.

 

Question 6

With reference to the period of Indian freedom struggle, which of the following was/were recommended by the Nehru report?

  1. Complete Independence for India.

  2. Joint electorates for reservation of seats for minorities

  3. Provision of fundamental right for the people of India in the constitution.

Select the correct answer using the codes given below:

(a) 1 only

(b) 2 and 3 only

(c) 1 and 3 only

(d) 1, 2 and 3

Ans: b

Sub-Theme: Nehru Report

Nehru Report (1928):

As an answer to Lord Birkenhead’s challenge, the Nehru report was prepared by a committee headed by Motilal Nehru, the committee included Tej Bahadur Sapru, Subhash Bose, M.S. Aney, Mangal Singh, Ali Imam, Shuab Qureshi and G.R. Pradhan as its members.

Recommendations:

  • Dominion status on lines of self- governing dominions. Therefore, statement 1 is incorrect.
  • Rejection of separate electorates. Joint electorates with reservation of seats for Muslims at the Centre and in provinces where they were in minority. Therefore, statement 2 is correct.
  •  Linguistic provinces.
    • Nineteen fundamental rights, including the rights to form unions, the right to equal treatment for women, and universal adult suffrage. Therefore, statement 3 is correct.
    • Responsible government at the Centre and in provinces.
    • Full protection of the cultural and religious interests of Muslims.
    • Complete dissociation of State from religion.
    • The three Indian Round Table Conference (1930–1932) attendees had access to both the Nehru Report and the Simon Commission report.
    • Nehru and Subash Bose rejected the congress goal and set up the Independence for India league.
NOTE: If we read NCERT of Modern India, we will come to know “Dominion Status” for India was the principal demand of the Nehru report. Just knowing this basic fact, we can eliminate option 1 and you will get the answer.

 

Question 7

Which one of the following observations is not true about the Quit India Movement of 1942?

(a) It was a non-violent movement

(b) It was led by Mahatma Gandhi

(c) It was a spontaneous movement

(d) It did not attract the labour class in general

Ans: b

Sub-Theme: Quit India Movement

Quit India Movement/August Revolution (1942):

  • After failure of Cripps’s Mission, a Resolution was framed by Gandhiji for British withdrawal and non-violent Non- Cooperation Movement against Japanese
  • Resolution was accepted in the CWC meeting in Wardha on July 14, 1942.
  • AICC Gowalia Tank Meeting, Bombay:
    • The Quit India Resolution was ratified in a Congress meeting at Gowalia Tank on 8th August 1942.
    • Apart from QIM, other resolutions involved the Civil Disobedience Movement against British Rule.
    • Mantra of “Do or Die” was adopted in this Session by Gandhiji.
    • Major activity after QIM resolution was passed:
    • It was started in response to Mahatma Gandhi’s national appeal for satyagraha. Gandhiji advocated for “an orderly British withdrawal” from India as the All-India Congress Committee announced a nationwide protest.
    • Destruction of symbols of Public Authority.
    • Underground activities: Usha Sharma started an underground Radio to give fuel to the movement.
    • Yusuf Meherally, a communist and trade unionist who also held the office of Mayor of Mumbai, is credited with coining the phrase “Quit India.”
    • Parallel Government: established in Ballia (UP), Tamluk (Bengal) organised “Bidyut Bahini”, Satara (Maharashtra) “Prati Sarkar” was organised by Y. B. Chavan, Nana Patil etc.
    •  Youth, Women, Workers, Peasants, Government officials especially of lower levels, Muslims, Communists participated in the movement. Upper class remained Loyalists.
    • There were no communal clashes during the movement. But there was severe government repression with upto 10,000 being killed.
  • Option (a) is correct: The movement basically promoted the non-violent and non-cooperative movement.
  • Option (b) is incorrect: The British Govt responded to the call of Gandhi by arresting all major Congress leaders the very next day. Gandhi, Nehru, Patel, etc. were all behind the bars. This left the movement in the hands of the younger leaders like Jayaprakash Narayan and Ram Manohar Lohia, besides many regional leaders also rose up to lead this movement in their respective regions.
  • Option (c) is correct: The Quit India movement was the spontaneous participation of the masses compared to the other Gandhian movements like non-cooperation and civil disobedience.
  • Option (d) is correct: Quit India Movement did not attract the labour class in general.
NOTE: Here we have to identify the option which is not true about the QIM.

 

Question 8

With reference to Indian freedom struggle, Usha Mehta is well-known for:

(a) Running the secret Congress Radio in the wake of Quit India Movement

(b) Participating in the second round Table Conference

(c) Leading a contingent of Indian National Army

(d) Assisting in the formation of Interim Government under Pandit Jawaharlal Nehru

Ans: a

Sub-Theme: Quit India Movement

Option (a) is correct: Usha Sharma started an underground Radio to give fuel to the movement. Hence, she was well-known for underground activities during the Quit India Movement.

Option (b) is incorrect: Second Round Table Conference was held in London between September 7, 1931 to December 1, 1931 and Sarojini Naidu participated to represent women.

Option (c) is incorrect: Usha Mehta was part of a small group which ran the Congress radio. She started an Underground Radio in Bombay whereas Captain Lakshmi Sahgal was the commander of the Jhansi Regiment of INA. Option (d) is incorrect: She was not a part of the Interim Government.

NOTE: It is always advisable to read in detail about the personalities involved in various movements. If we observe PYQs carefully, in every alternate year, there are personality based questions.

 Final Result – CIVIL SERVICES EXAMINATION, 2023.   Udaan-Prelims Wallah ( Static ) booklets 2024 released both in english and hindi : Download from Here!     Download UPSC Mains 2023 Question Papers PDF  Free Initiative links -1) Download Prahaar 3.0 for Mains Current Affairs PDF both in English and Hindi 2) Daily Main Answer Writing  , 3) Daily Current Affairs , Editorial Analysis and quiz ,  4) PDF Downloads  UPSC Prelims 2023 Trend Analysis cut-off and answer key

THE MOST
LEARNING PLATFORM

Learn From India's Best Faculty

      

 Final Result – CIVIL SERVICES EXAMINATION, 2023.   Udaan-Prelims Wallah ( Static ) booklets 2024 released both in english and hindi : Download from Here!     Download UPSC Mains 2023 Question Papers PDF  Free Initiative links -1) Download Prahaar 3.0 for Mains Current Affairs PDF both in English and Hindi 2) Daily Main Answer Writing  , 3) Daily Current Affairs , Editorial Analysis and quiz ,  4) PDF Downloads  UPSC Prelims 2023 Trend Analysis cut-off and answer key

Quick Revise Now !
AVAILABLE FOR DOWNLOAD SOON
UDAAN PRELIMS WALLAH
Comprehensive coverage with a concise format
Integration of PYQ within the booklet
Designed as per recent trends of Prelims questions
हिंदी में भी उपलब्ध
Quick Revise Now !
UDAAN PRELIMS WALLAH
Comprehensive coverage with a concise format
Integration of PYQ within the booklet
Designed as per recent trends of Prelims questions
हिंदी में भी उपलब्ध

<div class="new-fform">







    </div>

    Subscribe our Newsletter
    Sign up now for our exclusive newsletter and be the first to know about our latest Initiatives, Quality Content, and much more.
    *Promise! We won't spam you.
    Yes! I want to Subscribe.